You are on page 1of 88

1

Which of the following is considered most safe in pregnancy?


1

Benzoyl peroxide
2

Epinephrine
3

Methotrexate
4

Azelaic Acid
5

Isotretinoin
Q/Q(M)-482562 Report a Problem
Which of the following is considered most safe in pregnancy?
4

Azelaic Acid
Azelaic Acid is pregnancy category B. The other choices are in less safe pregnancy categories.
Epinephrine is category C, Benzoyl peroxide is category C, and Methotrexate and Isotretinoin are
category X.
Q/Q(M)-482562 Report a Problem
A patient presents with a likely fixed drug eruption. Her medications include glyburide, lisinopril,
hydrochlorthiazide, and aspirin, as well as an over-the-counter laxative. Which is the most likely culprit?
1

Glyburide
2

Lisinopril
3

Hydrochlorthiazide
4

Aspirin
5

Laxative
Q/Q(M)-475925 Report a Problem
A patient presents with a likely fixed drug eruption. Her medications include glyburide, lisinopril,
hydrochlorthiazide, and aspirin, as well as an over-the-counter laxative. Which is the most likely culprit?
5

Laxative
Phenopthalein, found in laxatives, is a known cause of fixed drug eruptions.
Q/Q(M)-475925 Report a Problem
A non-pigmenting fixed drug eruption is known to be caused by what agent?
1

Tetracycline
2

Barbiturates
3

Phenopthalein
4

Naproxen
5

Pseudoephedrine
Q/Q(M)-474460 Report a Problem
A non-pigmenting fixed drug eruption is known to be caused by what agent?
5

Pseudoephedrine
All agents listed are associated with fixed drug eruptions, however, pseudoephedrine hydrochloride is
the one most commonly associated with non-pigmenting fixed drug eruptions.
2

Q/Q(M)-474460 Report a Problem
Which of the following drugs is correctly matched to its target enzymes?
1

Tacrolimus-Thymidine kinase
2

Methotrexate-Inosine monophosphate dehydrogenase
3

Mycophenolate Mofetil-Phospholipase A2
4

Acyclovir-DNA polymerase
5

Terbinafine-Cytochrome p450
Q/Q(M)-478611 Report a Problem
Which of the following drugs is correctly matched to its target enzymes?
4

Acyclovir-DNA polymerase
Drug/Specific enzyme inhibited Tacrolimus - Calcineurin Methotrexate - Dihydrofolate reductase
Mycophenolate mofetil - Inosine monophosphate dyhydogenase Acyclovir - DNA Polymerase
Terbinafine - Squalene epoxidase
Q/Q(M)-478611 Report a Problem
Which of the following agents is NOT a UVB blocker?
1

Sulisobenzone
2

PABA
3

Cinnamates
4

Salicylates
5

Padimate A
Q/Q(M)-475902 Report a Problem
Which of the following agents is NOT a UVB blocker?
1

Sulisobenzone
Sulisobenzone is a benzophenone, a UVA blocker. The others are UVB blockers. Padimate A is a
PABA derivative.
Q/Q(M)-475902 Report a Problem
Which of the following is not a side effect of bleomycin?
1

Erythromelalgia
2

Pulmonary fibrosis
3

Flagellate hyperpigmentation
4

Serpentine supravenous hyperpigmentation
5

Acrosclerosis
Q/Q(M)-477725 Report a Problem
Which of the following is not a side effect of bleomycin?
4

Serpentine supravenous hyperpigmentation
All of the above are side effects of bleomycin except serpentine supravenous hyperpigmentation. This is
a side-effect of 5-fluorouracil.
Q/Q(M)-477725 Report a Problem
3


Which one of the following antifungals requires an acidic environment for its absorption?
1

amphotericin B
2

fluconazole
3

griseofulvin
4

terbinafine
5

itraconazole
Q/Q(M)-475859 Report a Problem
Which one of the following antifungals requires an acidic environment for its absorption?
5

itraconazole
Itraconazole is a triazole whose mechanism, like the other azoles, is inhibition of cytochrome P-450
enzyme lanosterol 14-alpha demethylase, with resultant inhibition of lanosterol conversion to ergosterol.
Ergosterol is an essential component of fungal cell membranes. Itraconazole is a weak base, which is
highly lipophilic and virtually insoluble in water. It is ionized only at a low pH. Griseofulvin is
administered in divided doses for the treatment of tinea capitus. Its absorption is optimized when given
with a fatty food. Oral fluconazole is very well absorbed when given orally with over 90%
bioavailability. Terbinafine, though highly lipophilic, has 70 to 80% absorption, when administered
orally, with a bioavailability not significantly impacted by food intake.
Q/Q(M)-475859 Report a Problem
Neutrophilic eccrine hidradenitis is associated most often with which chemotherapeutic agent?
1

5-fluorouracil
2

Cytarabine
3

Dactinomycin
4

Doxorubicin
5

Vinca alkaloids
Q/Q(M)-474471 Report a Problem
Neutrophilic eccrine hidradenitis is associated most often with which chemotherapeutic agent?
2

Cytarabine
Neutrophilic eccrine hidradenitis occurs in neutropenic patients with malignancies, usually acute
myelogenous leukemia. Cytarabine is the most commonly associated medication.
Q/Q(M)-474471 Report a Problem
A 10 year old child with a seizure disorder develops a morbiliform eruption and elevated LFT's two
weeks after starting Dilantin therapy. As his physician you:
1

Continue Dilantin and treat rash with topical corticosteroids
2

Discontinue Dilantin and begin Phenobarbital
3

Discontinue Dilantin and begin carbamazepine
4

Restart Dilantin once the rash resolves
5

Discontinue Dilantin and begin valproic acid
Q/Q(M)-477332 Report a Problem

4

A 10 year old child with a seizure disorder develops a morbiliform eruption and elevated LFT's two
weeks after starting Dilantin therapy. As his physician you:
5

Discontinue Dilantin and begin valproic acid
Anticonvulsant hypersensitivity syndrome (also drug rash with eosinophilia and systemic symptoms
(DRESS0 and dilantin hypersensitivity syndrome) presents with cutaneous eruption accompanied by
fever, facial edema, lymphadenopathy, leukocytosis and hepatitis. Cross reactivity is present in all
aromatic anticonvulsants including phenytoin, carbamazepine and phenobarbitol. There is no cross
reaction with valprioc acid.
Q/Q(M)-477332 Report a Problem
If a patient develops a dilantin hypersensitivity reaction, which anticonvulsant is the better alternative
therapy?
1

Carbamezapine
2

Phenytoin
3

Phenobarbital
4

Valproic acid
5

None of these answers are correct
Q/Q(M)-474123 Report a Problem
If a patient develops a dilantin hypersensitivity reaction, which anticonvulsant is the better alternative
therapy?
4

Valproic acid
Carbamezapine, phenytoin, and phenobarbital are known to cross-react with one another.
Q/Q(M)-474123 Report a Problem
Which of the following systemic corticosteroids has the highest glucocorticoid activity?
1

Betamethasone
2

Methylprednisolone
3

Triamcinolone
4

Cortisone
5

Hydrocortisone
Q/Q(M)-482746 Report a Problem
Which of the following systemic corticosteroids has the highest glucocorticoid activity?
1

Betamethasone
Systemic corticosteroids are commonly used in dermatology. Short acting steroids, cortisone and
hydrocortisone, have the greatest mineralocorticoid activity, while cortisone has the lowest
glucocorticoid activity. Intermediate and long-acting steroids, methylprednisolone, triamcinolone,
dexamethasone, and betamethasone, have virtually no mineralocorticoid activity. Dexamethasone and
betamethasone have the highest glucocorticoid activity.
Q/Q(M)-482746 Report a Problem
You see a patient with dark streaking in the nails, diffuse and oral hyperpigmented macules and
trichomegaly. Which of the following medication are they taking?
1

Imatinib
2

Zidovudine
5

3

Hydroxyurea
4

Cyclophosphamide
5

Bleomycin
Q/Q(M)-480140 Report a Problem
You see a patient with dark streaking in the nails, diffuse and oral hyperpigmented macules and
trichomegaly. Which of the following medication are they taking?
2

Zidovudine
All of the listed medications have been reported to cause nail hyperpigmentation, but the addition of oral
hyperpigmented macules and trichomegaly (lengthened anagen phase of the eyelashes). AZT can also
cause bone marrow suppression with subsequent anemia and granulocytopenia.
Q/Q(M)-480140 Report a Problem
Which chemical sunscreen has UVB and UVA II absorption capability?
1

Methyl anthranilate
2

Oxybenzone
3

Octyl salicylate
4

Titanium dioxide
5

Padimate O
Q/Q(M)-474462 Report a Problem
Which chemical sunscreen has UVB and UVA II absorption capability?
2

Oxybenzone
The benzophenones, oxybenzone and dioxybenzone, have the broadest absorption spectrum of the
chemical sunscreens, with UVB and UVA II range. Methyl anthranilate absorbs UVA II, and octyl
salicylate, and padimate O are UVB-absorbing chemicals. Titanium dioxide is not a chemical absorber,
it is a physical blocker.
Q/Q(M)-474462 Report a Problem
Which of the following retinoids is most lipophilic?
1

Etretinate
2

Acitretin
3

Tretinoin
4

Bexarotene
5

Isotretinoin
Q/Q(M)-482755 Report a Problem
Which of the following retinoids is most lipophilic?
1

Etretinate
Isotretinoin, acitretin, and bexarotene are water-soluble, with very little lipid deposition. Water-soluble
retinoids are undetectable in the serum 1 month after stopping therapy. Etretinate is 50 times more
lipophilic than acitretin. It can last several years in fatty tissues.
Q/Q(M)-482755 Report a Problem
Which of the following may cause an acneiform eruption?
1

Fluoxetine
6

2

Valproic acid
3

ACTH
4

Finasteride
5

Methotrexate
Q/Q(M)-474316 Report a Problem
Which of the following may cause an acneiform eruption?
3

ACTH
Many medications are associated with acneiform eruptions, including halogens (bromide and iodide),
androgenic hormones such as testosterone, ACTH, corticosteroids, isoniazid (INH), lithium, phenytoin,
and vitamins B2, B6 and B12.
Q/Q(M)-474316 Report a Problem
Which drug has been associated with cholestatic hepatitis?
1

Clindamycin
2

Erythromycin
3

Rifampin
4

Doxycycline
5

Ampicillin
Q/Q(M)-475198 Report a Problem
Which drug has been associated with cholestatic hepatitis?
2

Erythromycin
The estolate form of erythromycin has been associated with cholestatic hepatitis.
Q/Q(M)-475198 Report a Problem
What antihistamine can cause gynecomastia, impotence, and loss of libido?
1

Cyproheptadine
2

Chlorpheniramine
3

Cimetidine
4

Doxepin
5

Fexofendine
Q/Q(M)-474465 Report a Problem
What antihistamine can cause gynecomastia, impotence, and loss of libido?
3

Cimetidine
Cimetidine competitively inhibits dihydrotestosterone at the androgen receptor site and can exhibit anti-
androgen effects.
Q/Q(M)-474465 Report a Problem
Which of the following is NOT known to increase methotrexate levels?
1

Salicylates
2

Tetracyclines
3

Phenothiazines
7

4

NSAIDs
5

None of these answers are correct(all are known to increase methotrexate levels)
Q/Q(M)-475209 Report a Problem
Which of the following is NOT known to increase methotrexate levels?
5

None of these answers are correct(all are known to increase methotrexate levels)
Tetracyclines, phenytoin, phenothiazines, chloramphenicol, NSAIDs, salicylates, and sulfonamides,
among other drugs, can all increase methotrexate levels by displacement of plasma proteins.
Q/Q(M)-475209 Report a Problem
A patient with a long standing history of nodulocystic acne vulgaris on isotretinoin should avoid
tetracyclines with the drug to avoid complications such as:
1

Vitamine A exaggerated effect
2

Pseudotumor cerebri
3

Hepatotoxicity
4

Elevated triglycerides
5

Arthralgias
Q/Q(M)-482903 Report a Problem
A patient with a long standing history of nodulocystic acne vulgaris on isotretinoin should avoid
tetracyclines with the drug to avoid complications such as:
2

Pseudotumor cerebri
Isotretinoin is a oral retinoid that helps with nodulocystic acne. When isotretinoin is combined with
tetracycline there is an increased risk for pseudotumor cerebri. It should not be used together or in
patients with a history of pseudotumor cerebri that has been diagnosed by a neurologist. Pseudotumor
cerebri can be diagnosed by measuring the intraocular pressure, in which it will be elevated.
Q/Q(M)-482903 Report a Problem
How long after isotretinoin therapy can one safely begin trying to conceive?
1

Immediately
2

Two weeks
3

One month
4

One year
5

Three years
Q/Q(M)-477507 Report a Problem
How long after isotretinoin therapy can one safely begin trying to conceive?
3

One month
A woman should wait one month before trying to conceive after taking isotretinoin to prevent birth
defects. After taking acitretin a woman should wait three years before trying to conceive.
Q/Q(M)-477507 Report a Problem
What is the best medication to lower isotretinoin induced hypertriglyceridemia?
1

Simvastatin
8

2

Gemfibrozil
3

Niacin
4

Cholestyramine
5

All are equally effective
Q/Q(M)-476701 Report a Problem
What is the best medication to lower isotretinoin induced hypertriglyceridemia?
2

Gemfibrozil
Gemfibrozil generally reduces trygliceride levels to a greater extent than niacin, cholestyramine, and the
HMG-CoA reductase inhibitors.
Q/Q(M)-476701 Report a Problem
A patient who has been on dapsone for dermatitis herpetiformis develops cyanosis and appears
symptomatic. This patient is diagnosed with methemoglobinemia by his primary care doctor after doing
a complete blood count and hemoglobin. The deficiency that this patient most likely has is:
1

Glucose-6-phosphate dehydrogenase (G6PD)
2

Agranulocytosis
3

Aplastic Anemia
4

Pancytopenia
5

Xanthine Oxidase Deficiency
Q/Q(M)-482901 Report a Problem
A patient who has been on dapsone for dermatitis herpetiformis develops cyanosis and appears
symptomatic. This patient is diagnosed with methemoglobinemia by his primary care doctor after doing
a complete blood count and hemoglobin. The deficiency that this patient most likely has is:
1

Glucose-6-phosphate dehydrogenase (G6PD)
Dapsone is used in the treatment of dermatitis herpetiformis. Contraindications include severe
cardiovascular disease, marked renal insufficiency, and sulfonamide allergy. Patients need to be
screened for glucose-6-phosphate dehydrogenase (G6PD) deficiency. This is common among blacks and
Asians.
Q/Q(M)-482901 Report a Problem
Which of the following sclerotherapy agents can have a disulfiram like reaction?
1

polidocanol
2

sodium morrhuate
3

glycerin
4

hypertonic saline
5

sodium tetradecyl sulfate
Q/Q(M)-482919 Report a Problem
Which of the following sclerotherapy agents can have a disulfiram like reaction?
1

polidocanol
Polidocanol is a detergent type of sclera agent and can have a disulfiram like reaction. It also has a very
rare risk of anaphylaxis. Sodium morrhuate is also a detergent sclera agent and has the highest risk of
anaphylaxis.
Q/Q(M)-482919 Report a Problem
The medication most acceptable for usage in patients with renal failure is:
9

1

Tetracycline
2

Minocycline
3

Demeclocycline
4

Oxytetracycline
5

Doxycycline
Q/Q(M)-474118 Report a Problem
The medication most acceptable for usage in patients with renal failure is:
5

Doxycycline
Renal failure may prolong the half-life of most tetracyclines except doxycycline. Doxycycline is
excreted via the GI tract, unlike the other tetracyclines.
Q/Q(M)-474118 Report a Problem
Which of the following is a low sedation metabolite of hydroxyzine?
1

Fexofenadine
2

Cetirizine
3

Loratadine
4

Cyproheptadine
5

Ranitidine
Q/Q(M)-475911 Report a Problem
Which of the following is a low sedation metabolite of hydroxyzine?
2

Cetirizine
Cetirizine is a second-generation H1 antihistamine that is a low sedation metabolite of hydroxyzine.
Q/Q(M)-475911 Report a Problem
What is used to reduce bladder toxicity from cyclophosphamide?
1

Mesna
2

Leukovorin
3

Cimetidine
4

Vitamin E
5

Folic acid
Q/Q(M)-474452 Report a Problem
What is used to reduce bladder toxicity from cyclophosphamide?
1

Mesna
Mesna or sodium 2-mercptoethanesulfonate, has been used to reduce bladder toxicity from
cyclophosphamide.
Q/Q(M)-474452 Report a Problem
What is the difference between podophyllin and podophyllotoxin?
1

Essentially the same; they are interchangable in terms of treatment and side effects
2

Podophyllotoxin contains quercetin which is a potent mutagens
3

Podophyllin contains kaempherol which is a potent mutagens
10

4

Podophyllotoxin is a phosphodiesterase inhibitor
5

Podophyllin reversibly binds tubulin inhibiting cells in metaphase
Q/Q(M)-477637 Report a Problem
What is the difference between podophyllin and podophyllotoxin?
3

Podophyllin contains kaempherol which is a potent mutagens
Podophyllotoxin, also known as podofilox or Condolox, is a anti-mitotic agent that reversibly binds
tubulin, arresting cells in metaphase. It is used topically to treat genital warts. Podophyllin, which has
the same mechanism of action, contains kaempero and quercetin which are potent mutagens. Both are
derived from the May Apple plant. Cantharin, an antiviral agent derived from the Blister beetle, is a
phosphodiesterase inhibitior.
Q/Q(M)-477637 Report a Problem
Imiquimod is FDA approved for the treatment of which of the following?
1

Actinic keratosis
2

Squamous cell carcinoma in situ
3

Infiltrative basal cell carcinoma
4

Superficial spreading melanoma
5

Psoriasis vulgaris
Q/Q(M)-482821 Report a Problem
Imiquimod is FDA approved for the treatment of which of the following?
1

Actinic keratosis
Imiquimod is FDA approved for the treatment of actinic keratoses, superficial basal carcinomas, and
condyloma acuminate. It is not used to treat infiltrative basal cell carcinomas, squamous cell carcinoma
in situ, superficial spreading melanoma, or psoriasis. Off label uses include common warts, molluscum
contagiosum, Bowens disease (SCCIS), and keloids.
Q/Q(M)-482821 Report a Problem
Combination oral contraceptives decrease free testosterone levels by:
1

Directly binding free testosterone
2

Increasing SHBG (sex hormone binding globulin) production
3

Acting as competitive inhibitors of the androgen receptor
4

Acting as a GnRH agonist
5

Acting as a GnRH antagonist
Q/Q(M)-474322 Report a Problem
Combination oral contraceptives decrease free testosterone levels by:
2

Increasing SHBG (sex hormone binding globulin) production
Oral contraceptives decrease free testosterone levels by increasing the production of sex hormone
binding globulin (SHBG).
Q/Q(M)-474322 Report a Problem
Which of the following antifungal agent works by way of inhibiting ergosterol synthesis by blocking
squalene epoxidation:
1

Itraconazole
11

2

Terbinifine
3

Naftifine
4

Itraconazole and Terbinifine
5

Terbinifine and Naftifine
Q/Q(M)-478622 Report a Problem
Which of the following antifungal agent works by way of inhibiting ergosterol synthesis by blocking
squalene epoxidation:
5

Terbinifine and Naftifine
Terbinafine and Naftifine work by way of inhibiting ergosterol synthesis by blocking squalene
epoxidation (B&C). Itraconazole works by inhibiting ergosterol synthesis by blocking Lanosterol 14-
alpha demthylase.
Q/Q(M)-478622 Report a Problem
Anemia, leg ulcers, poikilodermatous skin changes, hepatitis, renal toxicity, and acral erythema are most
commonly associated with what medication?
1

5-fluorouracil
2

Hydroxyurea
3

Cyclosporine
4

Methotrexate
5

Doxorubicin
Q/Q(M)-474131 Report a Problem
Anemia, leg ulcers, poikilodermatous skin changes, hepatitis, renal toxicity, and acral erythema are most
commonly associated with what medication?
2

Hydroxyurea
The constellation of adverse effects is most closely associated with hydroxyurea.
Q/Q(M)-474131 Report a Problem
Which of the following are retinoid side effects?
1

Reversible hypothyroidism
2

Diffuse interstitial skeletal hyperostosis
3

Premature epiphyseal closure
4

Pseudotumor cerebri
5

All of these answers are correct
Q/Q(M)-475619 Report a Problem
Which of the following are retinoid side effects?
5

All of these answers are correct
Bexarotene has been shown to cause reversible hypothyroidism, not hyperthyroidism. Systemic retinoids
have been shown to cause diffuse interstitial skeletal hyperostosis, premature epiphyseal closure, and
pseudotumor cerebri (risk increased with concommitant use of tetracyclines).
Q/Q(M)-475619 Report a Problem
A 58-year old female receives thio-TEPA (trietheylenethiophosphoramide) for adenocarcinoma of the
breast. Which of the following cutaneous side effects might she expect?
12

1

Generalized hyperpigmentation sparing the palmar creases and mucous membranes
2

Hyperpigmentation of the teeth with permanent pigmentation of the gingival margin
3

Intense flushing of the skin
4

Hyperpigmentation of the axillae
5

Alternating colors of hair
Q/Q(M)-482270 Report a Problem
A 58-year old female receives thio-TEPA (trietheylenethiophosphoramide) for adenocarcinoma of the
breast. Which of the following cutaneous side effects might she expect?
4

Hyperpigmentation of the axillae
Thio-TEPA is an alkylating agent used in a variety of cancers. It can cause pruritus, urticaria,
angioedema, and hyperpigmentation localized to occluded areas. Generalized Addisonian-
hyperpigmentation may be caused by busulfan. Hyperpigmentation of the teeth with discoloration of the
gingival margins is associated with cyclophosphamide. Intense flushing of the skin is usually seen with
dacarbazine and carmustine. Alternating colors of hair, representing the "flag sign," can be seen with
methotrexate.
Q/Q(M)-482270 Report a Problem
Which of the following statements regarding antifungal medications is TRUE?
1

Griseofulvin is safe for patients with variegate porphyria
2

Ketoconazole has been associated with gynecomastia
3

Griseofulvin is a cytochrome P450 3A4 inhibitor
4

Terbinafine is fungistatic
5

Ketoconazole is fungicidal
Q/Q(M)-477568 Report a Problem
Which of the following statements regarding antifungal medications is TRUE?
2

Ketoconazole has been associated with gynecomastia
Medications that can precipitate acute attacks in variegate porphyria include barbiturates, estrogen,
griseofulvin, sulfonamides and ethanol. Ketoconazole can produce impotence and gynecomastia by
interfering with androgen synthesis. Ketoconazole inhibits cytochrome (CYP) P450 and most
concerning, can rarely cause fulminant hepatitis. Griseofulvin induces CYP P450, not inhibits it.
Terbinafine is fungicidal along with amphotericin B. Ketoconazole is fungistatic.
Q/Q(M)-477568 Report a Problem
The following drugs have been implicated in drug-induced subacute cutaneous lupus erythematosus:
1

Terbinafine
2

Verapamil
3

Pravastatin
4

All the above are correct
5

None of the above are correct
Q/Q(M)-477727 Report a Problem


The following drugs have been implicated in drug-induced subacute cutaneous lupus erythematosus:
13

4

All the above are correct
All of the above choices have been implicated in drug-induced subacute cutaneous lupus erythematosus.
Q/Q(M)-477727 Report a Problem

Which of the following medications is most likely to have caused this reaction?
1

Pseudoephedrine hydrochloride
2

Vancomycin
3

Lithium
4

ACE inhibitor
5

Diclofenac
Q/Q(M)-476869 Report a Problem


Which of the following medications is most likely to have caused this reaction?
1

Pseudoephedrine hydrochloride
Fixed drug eruptions occur 30 mintues to 8 hours after ingestion of offending agent. After rechallenge
with the same agent, the fixed drug eruption will recur. Potential causes of a fixed drug eruption include
analgesics, sulfonamides, barbituates, pseudoephedrine and anticonvulsants.
Q/Q(M)-476869 Report a Problem

Which site of the body has the highest penetration ability when topical medications applied
1

Face
2

Eyelids
3

Scrotum
4

Mucous membrane
5

Upper arms and legs
Q/Q(M)-482114 Report a Problem

Which site of the body has the highest penetration ability when topical medications applied
4

Mucous membrane
Drug penetration varies depending on body site and thickness of stratum corneum. An important
consideration in topical therapy is that abnormal skin may have an altered (increased, decreased, or
absent) stratum corneum, thus changing the body site's barrier function. Regional penetration of topical
therapy is ranked from highest penetration to less penetration as following: Mucous membrane >
scrotum > eyelids > face > chest and back > upper arms and legs > dorsal hands and feet > palms and
14

soles > nails
Q/Q(M)-482114 Report a Problem

The mechanism of action of mycophenolate mofetil most closely resembles that of what other drug?
1

Cyclophosphamide
2

Doxorubicin
3

Azathioprine
4

Hydroxyurea
5

Chlorambucil
Q/Q(M)-474127 Report a Problem

The mechanism of action of mycophenolate mofetil most closely resembles that of what other drug?
3

Azathioprine
Both azaithioprine and mycophenolate mofetil directly interfere with purine synthesis.
Q/Q(M)-474127 Report a Problem

Which drug has been associated with an increased incidence of serum sickness in children?
1

Rifampin
2

Clarithromycin
3

Ciprofloxacin
4

Clindamycin
5

Cefaclor
Q/Q(M)-475196 Report a Problem

Which drug has been associated with an increased incidence of serum sickness in children?
5

Cefaclor
Cefaclor has been associated with an increased incidence of serum sickness in children; the other drugs
have not.
Q/Q(M)-475196 Report a Problem

Which of the following drugs may cause acute generalized exanthematous pustulosis?
1

Ampicillin
2

Azithromycin
3

Cefazolin
4

Mercury
5

All of these answers are correct
Q/Q(M)-475924 Report a Problem

Which of the following medications would you not want a woman using an IUD as her only source of
contraception to take?
15

1

Ibuprofen
2

Cefalexin
3

Etanercept
4

Infliximab
5

Lidocaine
Q/Q(M)-480143 Report a Problem
Which of the following medications would you not want a woman using an IUD as her only source of
contraception to take?
1

Ibuprofen
NSAIDS (including ibuprofen) and Azathiprine have been associated with IUD failure. The other listed
medications are all pregnancy category B and should be safe for use in this patient.
Q/Q(M)-480143 Report a Problem
What is the treatment of choice for methemoglobinemia?
1

Methylene blue
2

Aspirin
3

Iron
4

Hydration
5

Observation
Q/Q(M)-476557 Report a Problem

What is the treatment of choice for methemoglobinemia?
1

Methylene blue
Methylene blue is reduced in the presence of NADPH and diaphorase II to leukomethylene blue, which
then reduces methemoglobin (Fe3+) to hemoglobin (Fe2+)
Q/Q(M)-476557 Report a Problem

The anti-HIV medication best known for causing a severe reaction which can result in fatality upon
rechallenge is:
1

Indinavir
2

Didanosine
3

Zidovudine
4

Abacavir
5

Nevirapine
Q/Q(M)-474124 Report a Problem

The anti-HIV medication best known for causing a severe reaction which can result in fatality upon
rechallenge is:
4

Abacavir
The hypersensivity reaction associated with abacavir usually resolves with cessation of the drug,
however upon rechallenge the reaction can be life-threatening.
16

Q/Q(M)-474124 Report a Problem

This retinoid targets RXR receptors:
1

Tretinoin
2

Isotretinoin
3

Acitretin
4

Bexarotene
5

Etretinate
Q/Q(M)-477674 Report a Problem

This retinoid targets RXR receptors:
4

Bexarotene
Bexarotene targets RXR receptors. It is used in the treatment of mycosis fungoides refractory to
conventional therapy. Side effects include central hypothyroidism and hyperlipidemia. These should be
treated with synthroid, lipitor, and fenofibrate.
Q/Q(M)-477674 Report a Problem

Peak vulnerability to thalidomide occurs between which days of gestation?
1

Days 1-14
2

Days 15-20
3

Days 21-36
4

Days 37-56
5

Days 57-70
Q/Q(M)-475920 Report a Problem
Peak vulnerability to thalidomide occurs between which days of gestation?
3

Days 21-36
Peak vulnerability to thalidomide occurs between days 21 to 36 of gestation, during which only a single
dose will cause birth defects to occur. Birth defects associated with thalidomide include phocomelia
(underdevelopment of arms and legs, the most common birth defect), ear malformation, and
gastrointestinal and urogenital defects.
Q/Q(M)-475920 Report a Problem
Which of the following drugs binds iron and thereby significantly prevents absorption?
1

Methotrexate
2

Azathioprine
3

Cyclosporine
4

Mycophenolate mofetil
5

Tacrolimus
Q/Q(M)-477437 Report a Problem

Which of the following drugs binds iron and thereby significantly prevents absorption?
17

4

Mycophenolate mofetil
Mycophenolate mofetil binds with Fe preparations preventing its absorption. Oral iron supplements
markedly reduce absorption of mycophenolate mofetil (CellCept). It is recommended that iron be
taken four to six hours before, or two hours after mycophenolate mofetil.
Q/Q(M)-477437 Report a Problem

The combination of ethanol and acitretin is potentially problematic because:
1

Ethanol exacerbates the cheilitis caused by acitretin
2

Acitretin increases the toxicity of ethanol
3

Ethanol inhibits the cytochrome p450 system
4

Ethanol promotes the conversion of acitretin to etretinate
5

Ethanol promotes the metabolism of acitretin
Q/Q(M)-476626 Report a Problem
The combination of ethanol and acitretin is potentially problematic because:
4

Ethanol promotes the conversion of acitretin to etretinate
Acitretin is an synthetic retinoid with affinity to the retinoic acid receptor (RAR). It can modulate the
proliferation and differentiation of epidermal keratinocytes. The concurrent injestion of ethanol and
alcohol increases the formation of etretinate. Unlike acitretin, etretinate is more lipophilic and
accumulates in the fat. It therefore has a much longer elimination half-life, estiimated at 120 days.
Q/Q(M)-476626 Report a Problem

Onycholysis is a potential side effect of treatment with which therapy?
1

Dapsone
2

Ketaconazole
3

Chloroquine
4

Tetracycline
5

Retinoids
Q/Q(M)-477201 Report a Problem

Onycholysis is a potential side effect of treatment with which therapy?
4

Tetracycline
Tetracycline has been associated with onycholysis and photo-onycholysis. Chloroquine may cause a
blue-brown discoloration of the nail bed and retinoids are associated with paronychia.
Q/Q(M)-477201 Report a Problem

Vascular leak syndrome has been associated with which chemotherapeutic agent?
1

Denileukin diftitox
2

Cytarabine
3

Methotrexate
4

Bleomycin
18

5

Interferon
Q/Q(M)-477151 Report a Problem
Vascular leak syndrome has been associated with which chemotherapeutic agent?
1

Denileukin diftitox
Denileukin difitox (brand name Ontak) is approved for the treatment of cutaneous T-cell lymphoma.
Denileukin difitox is a fusion protein composed from a portion of IL-2 with diphtheria toxin. The
chemotherapy is taken up by cells expressing high-affinity IL-2 receptors. Capillary leak syndrome
presenting with hypotension, edema, pleural effusions, and weight gain caused by fluid retention has
been reported in patients receiving denileukin difitox.
Q/Q(M)-477151 Report a Problem
Which antibiotic is the treatment of choice for pregnant patients with Rocky Mountain Spotted Fever?
1

Amoxicillin
2

Doxycycline
3

Erythromycin
4

Chloramphenicol
5

Trimethoprim
Q/Q(M)-482677 Report a Problem
Which antibiotic is the treatment of choice for pregnant patients with Rocky Mountain Spotted Fever?
4

Chloramphenicol
Chloramphenicol is the treatment of choice for pregnant patients with Rocky Mountain Spotted Fever.
In non-pregnant patients, the treatment of choice is Doxycycline.
Q/Q(M)-482677 Report a Problem
Which of the following is known to induce lichen planus-like eruptions?
1

Dapsone
2

Doxepin
3

Minocycline
4

Gold
5

Mercury
Q/Q(M)-474477 Report a Problem
Which of the following is known to induce lichen planus-like eruptions?
4

Gold
Mucocutaneous side effects of gold include stomatitis, cheilitis, lichen planus- like eruptions, and
pityriasis rosea-like eruptions.
Q/Q(M)-474477 Report a Problem
Which biologic agent is infused intravenously?
1

Alefacept
2

Etanercept
3

Efalizumab
4

Infliximab
5

None of these answers are correct
Q/Q(M)-475917 Report a Problem
19


Which biologic agent is infused intravenously?
4

Infliximab
Infliximab is infused intravenously.
Q/Q(M)-475917 Report a Problem

Which of the following events is most important in the pathogenesis of this painful eruption?
1

Increased expression of FasL
2

Reduction in circulating tumor necrosis factor
3

Reduction in circulating IL-6
4

Overexpression of keratins 6 and 16
5

Cleavage of desmoglein 1
Q/Q(M)-476748 Report a Problem


Which of the following events is most important in the pathogenesis of this painful eruption?
1

Increased expression of FasL
Toxic epidermal necrolysis is a life threatening drug eruption characterized by widespread epidermal
necrosis. The exact etiology of the keratinocyte necrosis has not been fully elucidated. However, FasL
(FasL and Fas are able to trigger apoptosis) has been shown to be upregulated in TEN.
Q/Q(M)-476748 Report a Problem
Which of the following supplements is most likely to decrease hemolysis associated in patients taking
dapsone?
1

Vitamin A
2

Vitamin B6
3

Folic acid
4

Vitamin D
5

Vitamin E
Q/Q(M)-476959 Report a Problem
Which of the following supplements is most likely to decrease hemolysis associated in patients taking
dapsone?
20

5

Vitamin E
Adverse effects from dapsone are both pharmacologic and idiosyncratic and include hemolytic anemia,
methemoglobinemia, agranulocytosis, hypersensitivity syndrome and neuropathy. Of these, the first two
are pharmacologic and anticipated, to some degree, in most patients treated with dapsone. However, the
magnitude of toxicity varies greatly among individuals on the drug. Methemoglobinemia is the
formation of methemoglobin in the blood, which has a decreased oxygen-carrying capacity compared
with hemoglobin and can result in cyanosis. The reaction is related to the N-hydroxy metabolites of
dapsone, which are potent oxidants. G6PD-deficient individuals are more susceptible to oxidative
stresses, including those from dapsone metabolites, and a baseline G6PD level is recommended prior to
initiation of dapsone therapy. Vitamin E (800 IU/day) has been shown to provide a small amount of
protection against methemoglobinemia and hemolysis, however, the clinical benefit of this strategy is
unclear.
Q/Q(M)-476959 Report a Problem
Which of the following class of medications has been associated with acquired brachial dyschromatosis?
1

Non-Steroidal Anti-inflammatory medications
2

ACE-inhibitors
3

Anti-virals
4

HMG-CoA Reductase inhibitors
5

Protease inhibitors
Q/Q(M)-477443 Report a Problem
Which of the following class of medications has been associated with acquired brachial dyschromatosis?
2

ACE-inhibitors
Acquired brachial dyschromatosis is a condition described as asymptomatic, gray-brown patches with
geographic borders, occasionally interspersed with hypopigmented macules, on the dorsum of the
forearms, mostly bilaterally and seen in middle aged women. Epidermal atrophy, basal layer
hyperpigmentation, elastosis and angiectases were histopathologic features. An association with
Civatte's poikiloderma as well as hypertension and/or antihypertensive drugs, especially ACE-inhibitors,
is suggested.
Q/Q(M)-477443 Report a Problem
Which antiparasitic agent acts by inhibiting fumarate reductase?
1

Ivermectin
2

Lindane
3

Permethrin
4

Thiabendazole
5

Cidofovir
Q/Q(M)-475909 Report a Problem
Which antiparasitic agent acts by inhibiting fumarate reductase?
4

Thiabendazole
Thiabendazole inhibits fumarate reductase, a helminth-specific enzyme. It is used to treat creeping
eruption or cutaneous larva migrans and larva currens. Ivermectin blocks glutamate-gated chloride ion
channels, and is used to treat strongyloidiasis, onchocerciasis, and Norwegian scabies. Lindane is an
organochloride which blocks neural transmission, and is effective against scabies, pubic lice, head lice,
and body lice. Permethrin disables sodium transport channels in the nerve cell membrane of the parasite.
21

Cidofovir is an antiviral nucleotide analogue.
Q/Q(M)-475909 Report a Problem
Which one of the following is not true about the mechanism of action or effects of glucocorticoids?
1

Decrease fibroblast production of collagen
2

Increase blood glucose
3

Decrease transcription of AP-1
4

Increase transcription of NF-kB
5

Form complexes with intracellular receptors
Q/Q(M)-482381 Report a Problem
Which one of the following is not true about the mechanism of action or effects of glucocorticoids?
4

Increase transcription of NF-kB
Glucocorticoids play a diverse role in the human body. They modulate transcription of specific genes
that lead to an increase or decrease in the levels of specific proteins, ie they decrease transcription of
AP-1 and NF-kB. All the other statements are true. Side effects include osteoporosis, hyperglycemia,
hypertension, poor wound healing, peptic ulcers, Cushingoid features, and muscle weakness.
Q/Q(M)-482381 Report a Problem
What drug is known to cause scotomas?
1

Dapsone
2

Isotretinoin
3

Quinacrine
4

Chloroquine
5

Gold
Q/Q(M)-474458 Report a Problem
What drug is known to cause scotomas?
4

Chloroquine
True retinopathy is associated with "bull's eye" pigment deposition, central scotoma, and diminished
visual acuity. Chloroquine is at greatest risk of causing retinopathy. Quinacrine is not associated with
the risk of retinopathy.
Q/Q(M)-474458 Report a Problem
Clofazamine hyperpigmentation has been described as:
1

Dark hyperpigmented streaks in the nails
2

Red-brown hyperpigmentation within skin lesions
3

Blue-gray hyperpigmentation over the anterior shins, palate, ears
4

Slate gray-purple hyperpigmentation in a photo-exposed distribution
5

Yellow discoloration of the skin, sclera
Q/Q(M)-474130 Report a Problem

Clofazamine hyperpigmentation has been described as:
2

Red-brown hyperpigmentation within skin lesions
Clofazamine can induce red-brown hyperpigmentation within skin lesions of patients with Hansen's
22

disease.
Q/Q(M)-474130 Report a Problem
What is the half-life of isotretinoin?
1

7 hours
2

20 hours
3

50 hours
4

30 days
5

120 days
Q/Q(M)-477613 Report a Problem

What is the half-life of isotretinoin?
2

20 hours
The half-life of isotretinoin is 20 hours. The half lives of bexarotene, acitretin, etretinate are 7 hours, 50
hours, 120 days respectively.
Q/Q(M)-477613 Report a Problem
Which of the following statements is NOT true regarding the categories for safety of drug use in
pregnancy?
1

Drugs are category A if controlled studies in humans show no risk to the fetus
2

Drugs are category B if controlled human studies show no risk to the fetus but may show risk to
animals, or if no risk has been shown in animal studies but no human studies have been
conducted
3

Drugs are category C if risk to the human fetus has been demonstrated, but animal studies are
equivocal
4

Drugs are category D if controlled studies show risk to human fetus, but in some instances
benefits may outweigh risks
5

Category X drugs are contraindicated in pregnancy
Q/Q(M)-474082 Report a Problem
Which of the following statements is NOT true regarding the categories for safety of drug use in
pregnancy?
3

Drugs are category C if risk to the human fetus has been demonstrated, but animal studies are
equivocal
Drugs are category C if risk to the human fetus cannot be ruled out, studies are lacking, or animal
studies are equivocal. Drugs for which risk to the human fetus has been demonstrated are pregnancy
category D.
Q/Q(M)-474082 Report a Problem

Which of the following pairings of antifungal agents and their mechanisms of action is NOT correct?
1

Terbinafine: Inhibits squalene epoxidase
2

Itraconazole: Inhibits 14-alpha-demethylase
3

Griseofulvin: Disrupts microtubule mitotic spindle formation
4

Ketoconazole: Blocks conversion of lanosterol to ergosterol
5

Fluconazole: Inhibits squalene epoxidase
Q/Q(M)-475206 Report a Problem
23

Which of the following pairings of antifungal agents and their mechanisms of action is NOT correct?
5

Fluconazole: Inhibits squalene epoxidase
Terbinafine inhibits squalene epoxidase and blocks the biosynthesis of ergosterol, a sterol essential to
the integrity of fungal cell membranes. Itraconazole inhibits 14-alpha-demethylase, blocking lanosterol
conversion to ergosterol. Griseofulvin disrupts microtubule mitotic spindle formation causing metaphase
arrest. Ketoconazole has a mechanism of action similar to itraconazole. Fluconazole also inhibits 14-
alpha-demethylase, not squalene epoxidase.
Q/Q(M)-475206 Report a Problem
A patient being treated for aspergillosis develops altered vision. What is the most likely medication he is
taking?
1

Caspofungin
2

Voriconazole
3

Amphotericin B
4

Itraconazole
5

Griseofulvin
Q/Q(M)-482462 Report a Problem
A patient being treated for aspergillosis develops altered vision. What is the most likely medication he is
taking?
2

Voriconazole
Voriconazole blocks ergosterol synthesis by inhibiting 14-alpha-demethylase. Side effects include visual
disturbances in 30%, headaches, nausea, vomiting, fever, and peripheral edema.
Q/Q(M)-482462 Report a Problem
Concomitant use of methotrexate and what other drug is contraindicated because of the potential
increased risk of pancytopenia?
1

Folic acid
2

NSAIDs
3

Tetracycline
4

Acetaminophen
5

Systemic retinoids
Q/Q(M)-474115 Report a Problem

Concomitant use of methotrexate and what other drug is contraindicated because of the potential
increased risk of pancytopenia?
2

NSAIDs
Drugs that simultaneously inhibit the folate metabolic pathway, such as NSAIDS, dapsone, or
trimethoprim-sulfamethoxazole, can increase hematologic toxicity when combined with methotrexate.
Q/Q(M)-474115 Report a Problem
Raynaud's phenomenon is a potential adverse effect of what chemotherapeutic agent?
1

5-fluorouracil
2

Doxorubicin
24

3

Bleomycin
4

Vinca alkaloids
5

Hydroxyurea
Q/Q(M)-474474 Report a Problem
Raynaud's phenomenon is a potential adverse effect of what chemotherapeutic agent?
3

Bleomycin
Bleomycin use has been associated with Raynaud's phenomenon occurring in digits treated with
intralesional therapy for periungual and plantar warts.
Q/Q(M)-474474 Report a Problem
Penile erosions are a reported side effect associated with which medication?
1

Trimethoprim-sulfamethoxazole
2

Acyclovir
3

Foscarnet
4

Azaithioprine
5

Bleomycin
Q/Q(M)-474126 Report a Problem

Penile erosions are a reported side effect associated with which medication?
3

Foscarnet
Genital ulcers have been reported as a complication of foscarnet therapy.
Q/Q(M)-474126 Report a Problem

Which of the following is correct regarding mycophenolate mofetil?
1

It is pregnancy category C drug
2

Metronidazole has been shown to increase the bioavailability of mycophenolate mofetil
3

It depletes the de novo production of guanosine nucleotides
4

It can be toxic in individuals with TPMT deficiency
5

The most common side effect is anemia.
Q/Q(M)-482520 Report a Problem
Which of the following is correct regarding mycophenolate mofetil?
3

It depletes the de novo production of guanosine nucleotides
Mycophenolate mofetil is a lymphocyte selective immunosuppressive agent that inhibits de novo purine
synthesis. Specifically, it depletes guanosine nucleotides by inhibiting inosine monophosphate
dehydrogenase. The most commonly reported side effects are GI and are dose-dependent.
Fluoroquinolones, rifampin, and metronidazole have been shown to decrease the bioavailability of
mycophenolate mofetil, which may result in lower circulating levels. Medications that result in elevated
concentrations of mycophenolate mofetil include salicylates and probenecid. There is also a possibility
of reduced concentration of nevirapine when coadministered with mycophenolate mofetil. It is currently
classified as FDA pregnancy category D. Azathioprine can be toxic in individuals with TPMT
deficiency, not mycophenolate mofetil.
Q/Q(M)-482520 Report a Problem
25

Finasteride is a specific inhibitor of:
1

Dihydrotesterone reductase
2

Type I 5 alpha reductase
3

Type II 5 alpha reductase
4

Aromatase
5

Testosterone synthetase
Q/Q(M)-478601 Report a Problem
Finasteride is a specific inhibitor of:
3

Type II 5 alpha reductase
Finasteride, a type II 5 alpha reductase inhibitor, given as a 1mg tablet daily, is effective in preventing
further hair loss and in increasing the hair counts to the point of cosmetically appreciatable results in
men ages 18 to 41 with mild to moderate hair loss at the vertex, in the anterior midscalp, and the frontal
region.
Q/Q(M)-478601 Report a Problem
The most common side effect of azathioprine is:
1

Bone marrow suppression
2

Neuropathy
3

Hepatotoxicity
4

Nephorotoxicity
5

Myopathy
Q/Q(M)-478603 Report a Problem
The most common side effect of azathioprine is:
1

Bone marrow suppression
The most common side effect of azathioprine (Imuran) is bone marrow suppresion (Pancytopenia). It is
a rare adverse event resulting from excessive immunosuppression by azathioprine. To prevent
catastrophic bone marrow failure, regular complete blood counts should be checked.
Q/Q(M)-478603 Report a Problem

Keratinocyte differentiation is enhanced by retinoids with all of the following EXCEPT:
1

Increased filaggrin production
2

Increased keratohyalin granules
3

Stimulation of ornithine decarboxylase
4

Odland body secretion of lipids
5

Increased keratin filaments
Q/Q(M)-475615 Report a Problem
Keratinocyte differentiation is enhanced by retinoids with all of the following EXCEPT:
3

Stimulation of ornithine decarboxylase
Keratinocyte differentiation is enhanced by retinoids with increased filaggrin production, increased
keratohyalin granules, keratin filaments, and Odland body secretion of lipids. Retinoids directly inhibit
ornithine decarboxylase and therefore lessen inflammatory hyperplasia.
26

Q/Q(M)-475615 Report a Problem
Which of the following biologic agents is pregnancy category C?
1

Alefacept
2

Infliximab
3

Efalizumab
4

Etanercept
5

All of these answers are correct
Q/Q(M)-475202 Report a Problem

Which of the following biologic agents is pregnancy category C?
3

Efalizumab
Efalizumab is pregnancy category C. The other drugs listed are pregnancy category B.
Q/Q(M)-475202 Report a Problem
Extension of this blister by application of perpendicular pressure is an example of what?
1

Fitzpatrick sign
2

Nikolsky sign
3

Asboe-Hansen's sign
4

Hutchinson's sign
5

Gorlin's sign
Q/Q(M)-476752 Report a Problem


Extension of this blister by application of perpendicular pressure is an example of what?
3

Asboe-Hansen's sign
Toxic epidermal necrolysis is a serious and potentially life threatening reaction to medications. The most
commonly implicated medications include penicillins, NSAIDS, and anti-convulsants. Asboe-Hansen's
sign results in extension of a blister with perpendicular pressure. Nikolsky sign is the separation of
epidermis from the dermis by application of tangential mechanical pressure.
27

Q/Q(M)-476752 Report a Problem
Which biologic agent is administered intramuscularly?
1

Alefacept
2

Efalizumab
3

Etanercept
4

Infliximab
5

None of the above
Q/Q(M)-475916 Report a Problem
Which biologic agent is administered intramuscularly?
1

Alefacept
Alefacept is given intramuscularly.
Q/Q(M)-475916 Report a Problem
What drug can potentiate bone marrow suppression when used concomitantly with azathioprine?
1

NSAIDS
2

Salicylates
3

Sulfonamides
4

Phenytoin
5

Allopurinol
Q/Q(M)-474456 Report a Problem
What drug can potentiate bone marrow suppression when used concomitantly with azathioprine?
5

Allopurinol
Concomitant allopurinol use, which inhibits xanthine oxidase, can lead to excess toxic purine analogs
via increased metabolism of azaithioprine via the HGPRT pathway, causing bone marrow suppression.
Q/Q(M)-474456 Report a Problem
A patient taking azathioprine for bullous pemphigoid develops a hypersensitivity syndrome
characterized by fever and shock. Approximately how long ago did the patient start this medication?
1

14 days
2

24 hours
3

3-4 days
4

6 weeks
5

1 week
Q/Q(M)-482750 Report a Problem
A patient taking azathioprine for bullous pemphigoid develops a hypersensitivity syndrome
characterized by fever and shock. Approximately how long ago did the patient start this medication?
1

14 days
Azathioprine is a purine analog which is used to treat multiple inflammatory disorders, including
autoimmune bullous disease, vasculitis, and severe dermatitis. A hypersensitivity syndrome with fever
and shock can occur at around 14 days.
Q/Q(M)-482750 Report a Problem
28

A patient is taking 15 mg of methotrexate each week for psoriasis. Which of the following medications
would be safe for use in this patient?
1

Tetracyclines
2

Phenytoin
3

Ibuprofen
4

Aspirin
5

Acetaminophen
Q/Q(M)-480141 Report a Problem
A patient is taking 15 mg of methotrexate each week for psoriasis. Which of the following medications
would be safe for use in this patient?
5

Acetaminophen
Methotrexate is a S-phase specific antimetabolite which competitively and irreversibly blocks
dihydrofolate reductase from catalyzing the formation of tetrahydrofolate, an important cofactor in
thymidylate and purine synthesis. Tetracyclines, phenytoin, phenothiazines, chloramphenicol, NSAIDs,
salicylates, and sulfonamides can all increase methotrexate levels by displacement of plasma proteins.
Acetaminophen is safe for concurrent use with methotrexate.
Q/Q(M)-480141 Report a Problem
Which of the following is not an ingredient of Castellani's paint?
1

Resorcinol
2

Industrial methylated spirit
3

Phenol
4

Boric Acid
5

Ethyl acetate
Q/Q(M)-476677 Report a Problem
Which of the following is not an ingredient of Castellani's paint?
5

Ethyl acetate
Castellani's paint was named after Sir Aldo Castellani and contains resorcinol, acetone, magenta, phenol,
boric acid, industrial methylated spirit, and water. It is fungicidal and bactericidal with local anesthetic
effects. It has been used to treat inflammatory tinea cruris, leg ulcers, and acute paronychia.
Q/Q(M)-476677 Report a Problem

Which of the following systemic agents has been shown to be the most effective in the treatment of
toenail onychomycosis?
1

Ketoconazole
2

Griseofulvin
3

Itraconazole
4

Fluconazole
5

Terbinafine
Q/Q(M)-477346 Report a Problem
Which of the following systemic agents has been shown to be the most effective in the treatment of
toenail onychomycosis?
29

5

Terbinafine
Craford et al. reviewed the available literature examining the efficacy of systemic anti-fungals and
performed a meta-analysis. Pooled analysis of cure rates at 11 and 12 months suggested that terbinafine
was more effective than itraconazole.
Q/Q(M)-477346 Report a Problem
The t 1/2 of isotretinoin is:
1

1 hour
2

7 hours
3

20 hours
4

2 days
5

120 days
Q/Q(M)-474327 Report a Problem
The t 1/2 of isotretinoin is:
3

20 hours
The t 1/2 of isotretinoin is 20 hours. The other answers list the t 1/2 times of various retinoids.
Q/Q(M)-474327 Report a Problem
For which of the following medications is sedation a very common side effect that may limit treatment?
1

Colchicine
2

Gold
3

Potassium iodide
4

Thalidomide
5

Chlorambucil
Q/Q(M)-475921 Report a Problem
For which of the following medications is sedation a very common side effect that may limit treatment?
4

Thalidomide
Sedation is a very common side effect of treatment with thalidomide. It is additive with other sedatives,
such as alcohol and barbiturates.
Q/Q(M)-475921 Report a Problem
Which of the following is a side effect of hydroquinone cream?
1

Telangiectasia
2

Photosensitivity
3

Ochronosis
4

Tachyphylaxis
5

Atrophy
Q/Q(M)-477171 Report a Problem
Which of the following is a side effect of hydroquinone cream?
3

Ochronosis
Exogenous ochronosis is an uncommon complication of irreversible pigmentation due to overuse of
30

topical hydroquinone (1,4 dihydroxybenzene). Hydroquinone acts to by melanocyte pigment production
by auto-oxidation of melanin, tyrosinase and phenol oxidases.
Q/Q(M)-477171 Report a Problem
A 52 year old male presents from home 5 days after excision of a basal cell carcinoma on his left shin
with pain, erythema, and a purulent discharge from the surgical site. He is placed on Cephalexin for a
total of 10 days. Two days later, he complains of worsening pain, redness, and discharge. Which of the
following is the best antibiotic choice?
1

Doxycycline
2

Amoxicillin
3

Azithromycin
4

Polymixin B
5

Ciprofloxacin
Q/Q(M)-482827 Report a Problem
A 52 year old male presents from home 5 days after excision of a basal cell carcinoma on his left shin
with pain, erythema, and a purulent discharge from the surgical site. He is placed on Cephalexin for a
total of 10 days. Two days later, he complains of worsening pain, redness, and discharge. Which of the
following is the best antibiotic choice?
1

Doxycycline
This patient presents with a surgical wound infection unresponsive to cephalexin. This is likely
secondary to community acquired MRSA infection. Doxycycline is the best choice for community
acquired MRSA infection. Bactrim, not listed, is another option. Amoxicillin, Azithromycin, and
Ciprofloxacin are not typically used to treat MRSA infections as resistance is high with these
medications. Polymixin B is a topical antibiotic with anti-pseudomonal properties.
Q/Q(M)-482827 Report a Problem
What is the mechanism utilized by the co-administration of probenicid to raise blood levels of
penicillins in patients with infections that require high blood levels?
1

Inhibition of cytochrome P-450 hepatic biotransformation system
2

Competitive inhibition of b-lactam binding sites
3

Displacement of plasma proteins
4

Synergistic effect of probenicid with penicillins
5

Prolongs the half-life of penicillins by decreasing renal tubular secretion
Q/Q(M)-474470 Report a Problem
What is the mechanism utilized by the co-administration of probenicid to raise blood levels of
penicillins in patients with infections that require high blood levels?
5

Prolongs the half-life of penicillins by decreasing renal tubular secretion
Probenicid is co-administered with penicillin to prolong its half-life through decreased renal tubular
secretion when higher blood levels are warranted.
Q/Q(M)-474470 Report a Problem
Which of the following medications is most likely to result in increased carbamazepine levels?
1

Minocycline
2

Rifampin
3

Erythromycin
31

4

Azithromycin
5

TMP-SMX
Q/Q(M)-474311 Report a Problem
Which of the following medications is most likely to result in increased carbamazepine levels?
3

Erythromycin
Eythromycin inhibits the hepatic cytochrome P450 system and can increase serum levels and potential
toxicities of carbamazapene, theophylline, warfarin, digoxin, methylprednisolone.
Q/Q(M)-474311 Report a Problem

What is the most likely congenital defect associated with isotretinoin therapy?
1

Atrial septal defect
2

Ventral septal defect
3

Cranial synostosis
4

Spina bifida
5

Phocomelia
Q/Q(M)-477657 Report a Problem
What is the most likely congenital defect associated with isotretinoin therapy?
3

Cranial synostosis
Isotretinoin is thought to cause congenital defects by interfering with neural crest development. The
most likely congenital defect is cranial synostosis.
Q/Q(M)-477657 Report a Problem

A patient takes oral clindamycin for a deep soft tissue infection due to a gram-positive organism. The
mechanism of action of clindamycin is most similar to which of the following antibiotics?
1

Erythromycin
2

Levofloxacin
3

Doxycycline
4

Penicillin
5

Rifampin
Q/Q(M)-482741 Report a Problem
A patient takes oral clindamycin for a deep soft tissue infection due to a gram-positive organism. The
mechanism of action of clindamycin is most similar to which of the following antibiotics?
1

Erythromycin
Clindamycin is particularly effective against anaerobic and gram-positive organisms, particularly those
causing deep tissue infections. It works by binding to the 50S ribosomal subunit thereby inhibiting
protein synthesis. Macrolide antibiotics such as erythromycin also work by this mechanism.
Fluoroquinolones block DNA gyrase. Tetracyclines inhibit the 30S subunit of ribosomes. Penicillin
blocks transpeptidation of the bacterial cell wall.
Q/Q(M)-482741 Report a Problem
Of the new biologic therapies for psoriasis, which agent requires weekly CD4 T-cell count monitoring?
32

1

Efalizumab
2

Alefacept
3

Infliximab
4

Etanercept
5

Adalimumab
Q/Q(M)-474461 Report a Problem
Of the new biologic therapies for psoriasis, which agent requires weekly CD4 T-cell count monitoring?
2

Alefacept
Alefacept is a human LFA-3/IgG fusion protein, which blocks LFA-3 on antigen presenting cells from
interacting with CD-2 on T-cells, preventing T-cell stimulation. Alefacept also eliminates activated
memory-effector T-cells, so weekly CD4 T-cell counts are required.
Q/Q(M)-474461 Report a Problem
Thalidomide is most associated with what adverse effect?
1

Sensory neuropathy
2

Distal motor neuropathy
3

Oral ulceration
4

Photosensitivity
5

Hypothyroidism
Q/Q(M)-474119 Report a Problem
Thalidomide is most associated with what adverse effect?
1

Sensory neuropathy
The most common presentation of the neuropathy from thalidomide is a mild proximal muscle weakness
with symmetric painful paresthesias of the distal extremities with accompanying lower limb sensory
loss. Hypothyroidism is a rarely reported adverse effect.
Q/Q(M)-474119 Report a Problem
Which of the following statements regarding podophyllin is NOT correct?
1

It is derived from the May apple plant
2

It binds tubulin
3

It arrests cells in telophase
4

It is contraindicated in pregnancy
5

None (all of these statements are true)
Q/Q(M)-475205 Report a Problem
Which of the following statements regarding podophyllin is NOT correct?
3

It arrests cells in telophase
Podophyllin is a crude cytotoxic extract from the May apple plant. It is antimitotic, arresting cells in
metaphase (not telophase) by binding to the protein tubulin. It may be teratogenic and should not be
used in pregnancy.
Q/Q(M)-475205 Report a Problem

The most common side effect of treatment with interferon-alpha is:
33

1

Weight loss
2

Nausea
3

Liver toxicity
4

Flu-like symptoms
5

Spastic diplegia
Q/Q(M)-477169 Report a Problem

The most common side effect of treatment with interferon-alpha is:
4

Flu-like symptoms
The most common side effect of treatment with interferon-alpha is flu-like symptoms of fever, chills,
myalgias, headache and arthralgias. Prophylactic administration of non steroidal anti-inflammatory
medications may alleviate some of these symptoms.
Q/Q(M)-477169 Report a Problem

Which of the following medications mechanism of action is through inhibition of DNA-dependent RNA
polymerase?
1

Penicillin G
2

Cephalexin
3

Tetracycline
4

Ciprofloxacin
5

Rifampin
Q/Q(M)-480138 Report a Problem

Which of the following medications mechanism of action is through inhibition of DNA-dependent RNA
polymerase?
5

Rifampin
Rifampin is an antibiotic used frequently for M. leprae and M. tuberculosis. It is the only drug
bactericidal to M. leprae. Its mechanism of action is by inhibiting RNA synthesis by inhibiting DNA-
dependent RNA polymerase. Penicillin G inhibits bacterial cell wall synthesis by blocking the
transpeptidation step. Cephalosporins have a similar mechanism of action, possessing a beta-lactam ring.
Tetracycline inhibits protein synthesis by binding and inhibiting the 30S ribosome while the
fluoroquinolone family inhibits DNA gyrase.
Q/Q(M)-480138 Report a Problem
Terfinadine is an antihistamine that has been used in the past. However, it is no longer available in the
USA due to issues with :
1

Life-threatening cardiac arrhythmias
2

Thyroid cancer
3

Seizures
4

Cyanosis
5

Agranulocytosis
Q/Q(M)-482902 Report a Problem
34

Terfinadine is an antihistamine that has been used in the past. However, it is no longer available in the
USA due to issues with :
1

Life-threatening cardiac arrhythmias
Terfenadine is an anti-histamine that was one of the first nonsedating antihistamines. It is no longer
available in the USA because of its propensity to cause life-threatening cardiac arrhythmias. The
problem is worsen by interaction with macrolide antibiotics and imidazole antifungals.
Q/Q(M)-482902 Report a Problem

Which of the following is true of the live attenuated varicella vaccine when given to healthy adults?
1

It was marginally effective
2

It provides only short term protection (6 months)
3

It protects even individuals who never had serconversion or whose antibody levels were
undetectable from severe varicella zoster viral disease
4

The disease process will be accelerated
5

Disseminated zoster is frequently seen
Q/Q(M)-478739 Report a Problem
Which of the following is true of the live attenuated varicella vaccine when given to healthy adults?
3

It protects even individuals who never had serconversion or whose antibody levels were
undetectable from severe varicella zoster viral disease
When live attenuated varicella vaccine is given to healthy adults, it protects even individuals who never
had serconversion or whose antibody levels were undetectable from severe varicella zoster viral disease.
Q/Q(M)-478739 Report a Problem
A middle aged gentleman with pemphigus vulgaris is managed with azathioprine. Which of the
following substances is the active metabolite?
1

6-thioguanine
2

Azathioprine
3

6-mercaptopurine
4

Thiouric acid
5

6-thioinosine monophosphate
Q/Q(M)-482749 Report a Problem

A middle aged gentleman with pemphigus vulgaris is managed with azathioprine. Which of the
following substances is the active metabolite?
1

6-thioguanine
Azathioprine is a purine analog that is used as a steroid-sparing agent in a variety of dermatologic
inflammatory disorders. Azathioprine is converted into 6-mercaptopurine before being converted into
the active metabolite, 6-thioguanine, by the hypoxanthine guanine phosphoribosyltransferase pathway.
Other inactive metabolites are produced via xanthine oxidase and thiopurine methyltransferase.
Q/Q(M)-482749 Report a Problem
The anti-viral agent used most often for acyclovir-resistent HSV and VZV infections is:
1

Valacyclovir
35

2

Gancyclovir
3

Foscarnet
4

Famcyclovir
5

Pencyclovir
Q/Q(M)-474472 Report a Problem

The anti-viral agent used most often for acyclovir-resistent HSV and VZV infections is:
3

Foscarnet
Foscarnet does not require phosphorylaton for antiviral activity. Thereofre, it is achieve against viruses
resistant to acyclovir, famcyclovir, or gancyclovir on the basis of altered-kinase activities.
Q/Q(M)-474472 Report a Problem
Potentially fatal ventricular arrhythmias can occur with concomitant use of cisapride and:
1

Astemizole
2

Digoxin
3

Terbinafine
4

Erythromycin
5

Atorvostatin
Q/Q(M)-474309 Report a Problem
Potentially fatal ventricular arrhythmias can occur with concomitant use of cisapride and:
4

Erythromycin
Co-administration of erythromycin with the antihistamines terfenedine and astemizole or the
gastrointestinal promobility agent cisapride increases the risk of torsade de pointes and is
contraindicated. These drugs are no longer available in the US.
Q/Q(M)-474309 Report a Problem

A patient with acute diarrhea is prescribed antibiotic treatment for his symptoms. He subsequently
suffers from nausea and vomiting after ingesting alcohol. What is the most likely medication he is
taking?
1

Ciprofloxacin
2

Azithromycin
3

Penicillin
4

Clindamycin
5

Metronidazole
Q/Q(M)-477673 Report a Problem
A patient with acute diarrhea is prescribed antibiotic treatment for his symptoms. He subsequently
suffers from nausea and vomiting after ingesting alcohol. What is the most likely medication he is
taking?
5

Metronidazole
This patient is most likely taking metronidazole for acute diarrhea secondary to giardella. Metronidazole
causes antabuse-like reactions with ingestion of alcohol.
Q/Q(M)-477673 Report a Problem
36


Which of the following is the most common adverse effect of Thalidomide therapy?
1

Diarrhea
2

Sedation
3

Skin discoloration
4

Photosensitivity
5

Hypertension
Q/Q(M)-478627 Report a Problem

Which of the following is the most common adverse effect of Thalidomide therapy?
2

Sedation
Thalidomide was introduced in the late 1950's as a "safe" sleeping aide. It readily penetrates the CNS,
where it exerts a hyposedative effect comparable with barbiturates. By far, the most common adverse
effect from thalidomide is sedation, which in many patients may require that primarily night-time doses
be utilized.
Q/Q(M)-478627 Report a Problem
Which of the following biologic therapies is pregnancy category C?
1

Etanercept
2

Alefacept
3

Infliximab
4

Efalizumab
5

Adalimumab
Q/Q(M)-477654 Report a Problem

Which of the following biologic therapies is pregnancy category C?
4

Efalizumab
All of the above drugs are pregnancy category B except efalizumab which is category C.
Q/Q(M)-477654 Report a Problem
Which of the following statements regarding drug interactions is true?
1

Drugs that induce CYP3A enzymes may decrease levels of drugs which act as substrates for
CYP3A
2

CYP3A inhibitors may increase levels and cause toxicity of drugs metabolized by cytochrome P-
450
3

Terbinafine is not metabolized by cytochrome P-450
4

Tobacco induces P-450 enzymes
5

All of the above are true
Q/Q(M)-475923 Report a Problem

Which of the following statements regarding drug interactions is true?
5

All of the above are true
The most relevant drug interactions in dermatology involve the hepatic biotransformation pathways
37

catalyzed by the cytochrome P-450 isoenzymes from the subfamilies CYP3A3/4. Drugs that induce
CYP3A enzymes may decrease levels of drugs which act as substrates for CYP3A. CYP3A inhibitors
may increase levels and cause toxicity of drugs metabolized by cytochrome P-450. Terbinafine is not
metabolized by cytochrome P-450, but inhibits CYP2D6-mediated metabolism. Tobacco induces P-450
enzymes.
Q/Q(M)-475923 Report a Problem

In addition to minocycline, which of the following drugs has been associated with drug-induced lupus
erythematosus-like syndrome?
1

Amiodarone
2

Itraconazole
3

Rifampin
4

Doxycycline
5

Hydralazine
Q/Q(M)-474317 Report a Problem

In addition to minocycline, which of the following drugs has been associated with drug-induced lupus
erythematosus-like syndrome?
5

Hydralazine
Drugs associated with drug-induced SLE include minocycline, hydralazine, procainamide, isonaizid,
penicillamine, and anti-convulsants.
Q/Q(M)-474317 Report a Problem
A female patient with adult acne is place on spironolactone for her acne vulgaris. As a physician, you
warn her that there is a black box warning against patients with a :
1

Personal history of breast cancer
2

Personal history of thyroid cancer
3

Personal history of bladder cancer
4

Personal history of colon cancer
5

Personal history of rectal cancer
Q/Q(M)-482899 Report a Problem

A female patient with adult acne is place on spironolactone for her acne vulgaris. As a physician, you
warn her that there is a black box warning against patients with a :
1

Personal history of breast cancer
Spironolactone is an antiandrogenic potassium sparing diuretic. It can be used for women with adult
acne that is recalcitrant to treatment. Although it is not first line, it helps to block androgens that cause
acne in the jawline. Spironolactone has a black box warning against patient with a personal history of
breast cancer or family history of breast cancer.
Q/Q(M)-482899 Report a Problem

Approximately what percentage of patients with drug hypersensitivity syndrome will have liver function
test abnormalities?
1

Less than 10%
38

2

25%
3

50%
4

75%
5

Close to 100%
Q/Q(M)-474318 Report a Problem

Approximately what percentage of patients with drug hypersensitivity syndrome will have liver function
test abnormalities?
3

50%
Drug hypersensitivity syndrome is characterized by fever, skin eruption and internal organ involvement.
Drugs associated with drug hypersensitivity syndrome include sulfonamindes, dapsone, anticonvulsants
(carbamezapine, phenobarbitol, lamotrigine), anti-retrovirals (ritonovir, nevirapine) and minocycline.
Approximately 50% of patients will have abnormal liver enzymes.
Q/Q(M)-474318 Report a Problem
A patient is being treated by a dermatologist. Yellowish changes of his sclera and yellowish
discoloration of his dorsal hands and feet is noted. Which medication is most likely responsible?
1

Quinacrine
2

Minocycline
3

Hydroxychloroquine
4

Terbinafine
5

Prednisone
Q/Q(M)-482753 Report a Problem

A patient is being treated by a dermatologist. Yellowish changes of his sclera and yellowish
discoloration of his dorsal hands and feet is noted. Which medication is most likely responsible?
1

Quinacrine
Quinacrine frequently produces a yellow discoloration of the sclera and skin, especially over the dorsal
hands and feet. Minocycline and hydroxychloroquine can cause bluish-gray hyperpigmentation.
Terbinafine and prednisone are not associated with alteration of cutaneous pigmentation.
Q/Q(M)-482753 Report a Problem

The Mazzotti reaction is most closely associated with administration of which of the following
medications?
1

capecitabine
2

diethylcarbamazine
3

penicillamine
4

cisplatin
5

ipilimumab
Q/Q(M)-482830 Report a Problem

The Mazzotti reaction is most closely associated with administration of which of the following
medications?
39

2

diethylcarbamazine
The Mazzotti reaction is characterized by urticaria, fever, chills, sweats, abdominal pain, arthralgias, and
lymphadenopathy. Hypotension and tachycardia may also be present, and the reaction can be life
threatening. Mazzotti reactions are most often associated with the administration of diethylcarbamazine
for the treatment of onchocerciasis, but may be seen during treatment with other anti-helminthics, such
as suramin. The pathogenesis of Mazzotti reactions is thought to be consequent to abrupt release of
parasitic antigens during cell death, prompting an intense inflammatory cascade. Currently, ivermectin is
the treatment of choice for onchocerciasis, which has a lower incidence of associated Mazzotti reactions
(approximately 10%).
Q/Q(M)-482830 Report a Problem

Terbinafine exerts its antifungal activity by what manner?
1

Inhibition of 14-a demethylase
2

Inhibition of squalene epoxidase
3

Inhibition of epoxide hydroxylase
4

Interference with cell respiratory processes
5

Direct binding to membrane sterols, increasing permeability
Q/Q(M)-474121 Report a Problem

Terbinafine exerts its antifungal activity by what manner?
2

Inhibition of squalene epoxidase
Terbinafine, an allylamine, interferes with ergosterol synthesis by inhibiting squalene epoxidase. The
azoles inhibit 14-a demethylase. Nystatin is a polyene which binds irreversibly to membrane sterols,
resulting in a permeability shift. Ciclopirox does not appear to affect sterol biosynthesis but instead
interferes with cell respiratory processes.
Q/Q(M)-474121 Report a Problem

Which of the following chemotherapeutic agents causes increased growth of eyelashes?
1

Interleukin 2
2

Interferon alpha
3

Mitomycin
4

Cytarabine
5

Methotrexate
Q/Q(M)-476709 Report a Problem

Which of the following chemotherapeutic agents causes increased growth of eyelashes?
2

Interferon alpha
Interferons can cause increased growth of eyelashes. Trichomegaly has been reported after treatment
with interferon-alpha in patients with chronic hepatitis, B-cell lymphoma, chronic granulocytic
leukemia, and cutaneous melanoma. Trichomegaly has also been reported in associatoin with
latanoprost, minoxidil, cyclosporine, phenytoin, psoralen, and penicillamine.
Q/Q(M)-476709 Report a Problem

Which chemotherapeutic agent has been reported to cause acral sclerosis with Raynaud's phenomenon?
40

1

Bleomycin
2

5-Fluorouracil
3

Methotrexate
4

Actinomycin
5

Interferon
Q/Q(M)-476719 Report a Problem
Which chemotherapeutic agent has been reported to cause acral sclerosis with Raynaud's phenomenon?
1

Bleomycin
Bleomycin is an antibiotic that induces single strand breaks in the DNA. Reactions to bleomycin include
flagellae hyperpigmentation, acral sclerosis with Reynaud's, penile calcification, and a morbilliform
eruption.
Q/Q(M)-476719 Report a Problem

Cefaclor has been associated with increased incidence of what in children?
1

Mononucleosis-like syndrome
2

Anaphylaxis
3

Transaminitis
4

Serum sickness reaction
5

Generalized tonic-clonic seizures
Q/Q(M)-474451 Report a Problem

Cefaclor has been associated with increased incidence of what in children?
4

Serum sickness reaction
The use of cefaclor has been associated with an increased incidence of serum sickness in children.
Q/Q(M)-474451 Report a Problem

What medication's mechanism of action is via suppression of the halide-myeloperoxidase system?
1

Cyclophosphamide
2

Quinicrine
3

Dapsone
4

Melphalan
5

Hydroxyurea
Q/Q(M)-477116 Report a Problem
What medication's mechanism of action is via suppression of the halide-myeloperoxidase system?
3

Dapsone
Dapsone inhibits the neutrophil halide-myeloperoxidase system which results in an impaired respiratory
burst and subsequent tissue damage.
Q/Q(M)-477116 Report a Problem

The anti-CD 20 antibody rituximab is FDA-approved for treatment of which of the following?
41

1

Mycosis fungoids
2

Metastatic melanoma
3

Psoriasis
4

Paraneoplastic pemphigus
5

Non-Hodgkins lymphoma
Q/Q(M)-477154 Report a Problem

The anti-CD 20 antibody rituximab is FDA-approved for treatment of which of the following?
5

Non-Hodgkins lymphoma
Rituximab (brand name Rituxan) is a monoclonal antibody is approved for the treatment of CD20 non-
Hodgkins lymphoma. Rituximab is a monoclonal antibody directed against B lymphocytes which are
CD20.
Q/Q(M)-477154 Report a Problem
All of the following topical antioxidants have demonstrated cutaneous anticarcinogenic effects in mice
except:
1

Zinc
2

Vitamin C
3

Tea polyphenois
4

Vitamin E
5

Silymarin
Q/Q(M)-477218 Report a Problem

All of the following topical antioxidants have demonstrated cutaneous anticarcinogenic effects in mice
except:
1

Zinc
Anti-oxidants are thought to be protective against photoinjury by neutralizing oxygen radicals. Vitamin
C, Vitamin E, tea polyphenois, and silymarin are all anti-oxidants.
Q/Q(M)-477218 Report a Problem
What antifungal is the best choice for a patient with mucocutaneous candidiasis who is currently taking
antacids?
1

Itraconazole
2

Fluconazole
3

Ketoconazole
4

Terbinafine
5

Griseofulvin
Q/Q(M)-474473 Report a Problem

What antifungal is the best choice for a patient with mucocutaneous candidiasis who is currently taking
antacids?
2

Fluconazole
42

Ketoconazole and itraconazole require an acidic environment. Fluconazole, however, does not require
an acidic environment and can work safely and effectively in patients taking antacids, which can raise
gastric pH levels.
Q/Q(M)-474473 Report a Problem

Which of the following is the highest potency topical corticosteroid?
1

Halobetasol propionate cream 0.05%
2

Triamcinolone acetonide ointment 0.1%
3

Fluocinolone acetonide cream 0.025%
4

Hydrocortisone valerate ointment 0.2%
5

Mometasone furoate ointment 0.1%
Q/Q(M)-482833 Report a Problem

Which of the following is the highest potency topical corticosteroid?
1

Halobetasol propionate cream 0.05%
Halobetasol propionate cream 0.05% is a superpotent Class 1 topical steroid. Triamcinolone acetonide
ointment 0.1% and hydrocortisone valerate ointment 0.2% are medium potency class 4 topical steroids.
Fluocinolone acetonide cream 0.025% is a medium potency class 4 topical steroid. Mometasone furoate
ointment 0.1% is a high potency class 2 topical steroid.
Q/Q(M)-482833 Report a Problem

Which of the following medications would be safe for use in pregnancy?
1

Tazarotene
2

Finasteride
3

Etretinate
4

Azeleic acid
5

5-fluorouracil topically
Q/Q(M)-480142 Report a Problem
Which of the following medications would be safe for use in pregnancy?
4

Azeleic acid
All of the listed medications except azeleic acid are pregnancy category X. Azeleic acid is a pregnancy
category B medication, meaning that there are controlled human studies that show no risk to fetus but
may show risk to animals, or no risk in animals with no human studies conducted.
Q/Q(M)-480142 Report a Problem
Painful periungual pyogenic granulomas have been associated with what medication?
1

Ketoconazole
2

Indinavir
3

Doxycycline
4

Tazarotene
5

Valacyclovir
Q/Q(M)-474125 Report a Problem
Painful periungual pyogenic granulomas have been associated with what medication?
43

2

Indinavir
Periungual pyogenic granulomas and painful paronychial eruptions have been reported in association
with various anti-HIV medications including, indinavir, zidovudine, and lamivudine.
Q/Q(M)-474125 Report a Problem
Fomivirsen is a single-stranded antisense oligonucleotide FDA-approved for the treatment of:
1

HSV infection
2

HIV infection
3

EBV infection
4

CMV infection
5

HPV infection
Q/Q(M)-477212 Report a Problem

Fomivirsen is a single-stranded antisense oligonucleotide FDA-approved for the treatment of:
4

CMV infection
Fomiversen (Vitraene) represents a new class of therapeutic agents known as antisense drugs. It is
indicated for the treatment of CMV retinitis in patients with AIDS. The most common adverse effects
are ocular inflammation and increases in intraocular pressure.
Q/Q(M)-477212 Report a Problem
Concomitant administration of retinoid or ketoconazole with doxorubicin may lead to which side effect?
1

Dermatomyositis-like reaction
2

Papulopustular eruption
3

Capillary leak syndrome
4

Sticky skin
5

Leg ulcers
Q/Q(M)-482271 Report a Problem

Concomitant administration of retinoid or ketoconazole with doxorubicin may lead to which side effect?
4

Sticky skin
Doxorubicin when given with retinoids or ketoconazole may lead to sticky skin. A dermatomyositis-like
reaction and leg ulcers may be seen with hydroxyurea. Epidermal growth factor receptors such as
erlotinib and cetuximab can produce papulopustular eruptions. Capillary leak syndrome is seen with
interleukins.
Q/Q(M)-482271 Report a Problem

What tetracycline is not phototoxic?
1

Minocycline
2

Doxycycline
3

Oxytetracycline
4

Tetracycline
5

Demeclocycline
44

Q/Q(M)-474475 Report a Problem
What tetracycline is not phototoxic?
1

Minocycline
Minocycline is not phototoxic. Demeclocycline and doxycycline are the most phototoxic of all the
tetracyclines. Onycholysis can accompany tetracycline-induced phototoxicity.
Q/Q(M)-474475 Report a Problem
All of the following are reported cutaneous side effects of zidovudine EXCEPT:
1

Trichomegaly
2

Diffuse and oral hyperpigmented macules
3

Hyperpigmented streaks in nails
4

Periungual pyogenic granulomas
5

None of the above (all are reported side effects)
Q/Q(M)-475204 Report a Problem
All of the following are reported cutaneous side effects of zidovudine EXCEPT:
5

None of the above (all are reported side effects)
All of the following are cutaneous side effects reported with zidovudine, a nucleoside HIV reverse
transcriptase inhibitor. Periungal/paronychial eruptions resulting in pyogenic granuloma-like lesions
have also been reported with other HIV medications, including indinavir and lamivudine.
Q/Q(M)-475204 Report a Problem
What is the target antigen for rituximab?
1

CD4
2

CD7
3

CD8
4

CD20
5

CD22
Q/Q(M)-477742 Report a Problem
What is the target antigen for rituximab?
4

CD20
Rituximab is an anti-CD20 monoclonal antibody. CD20 is a B-cell marker and is used to treat Non-
Hodgkins B-cell lymphoma. It is also approved for the treatment of rheumatoid arthritis.
Q/Q(M)-477742 Report a Problem
What drug can induce native SLE?
1

Hydralazine
2

Procainamide
3

Isoniazide
4

TNF inhibitor
5

Minocycline
Q/Q(M)-482851 Report a Problem
What drug can induce native SLE?
4

TNF inhibitor
45

Drug induced lupus consists of systemic lupus type symptoms with anti-histone antibodies. Common
culprit drugs are hydralazine, procainamide, isoniazid, and minocycline. Cessation of the medication
usually results in improvement of symptoms. The TNF inhibitors also induce systemic lupus, but this
typically is an unmasking of native lupus that is not associated with anti-histone antibodies and does not
remit with cessation of treatment.
Q/Q(M)-482851 Report a Problem

The mechanism of action of podophyllin most closely resembles that of what other drug listed below?
1

Chlorambucil
2

Dactinomycin
3

Colchicine
4

5-fluorouracil
5

Permethrin
Q/Q(M)-474476 Report a Problem
The mechanism of action of podophyllin most closely resembles that of what other drug listed below?
3

Colchicine
Both podophyllin and colchicine have antimitotic activity. They bind to tubulin dimers, interfering with
mitotic spindle and microtubule assembly.
Q/Q(M)-474476 Report a Problem
Which antiviral agent has been associated with fatal thrombotic thrombocytopenic purpura in AIDS and
transplant patients taking high doses?
1

Acyclovir
2

Valacyclovir
3

Famciclovir
4

Penciclovir
5

Cidofovir
Q/Q(M)-475203 Report a Problem
Which antiviral agent has been associated with fatal thrombotic thrombocytopenic purpura in AIDS and
transplant patients taking high doses?
2

Valacyclovir
Valacyclovir has been associated with severe and even fatal cases of thrombotic thrombocytopenic
purpura / HUS syndrome in AIDS and transplant patients taking high doses.
Q/Q(M)-475203 Report a Problem
A patient has a single oval erythematous patch on his abdomen. Biopsy reveals pigment incontinence
and basal vacuolization. A diagnosis of fixed drug eruption is made. What is the most common cause of
fixed drug eruption?
1

Tetracycline
2

NSAIDs
3

Phenylephrine
4

Amoxicillin
5

Simvastatin
46

Q/Q(M)-482739 Report a Problem
A patient has a single oval erythematous patch on his abdomen. Biopsy reveals pigment incontinence
and basal vacuolization. A diagnosis of fixed drug eruption is made. What is the most common cause of
fixed drug eruption?
1

Tetracycline
A fixed drug eruption is a single or multiple oval or annular erythematous patches that arise as a result
of exposure to a systemic medication. Repeat exposure to the offending drug can cause reappearance of
the lesions in the same location. Tetracycline is the most common cause of fixed drug eruption.
Q/Q(M)-482739 Report a Problem
A patient presents with tingling and burning of their lateral upper lip. They report that blisters will form
which then crust and heal. She gets these episodes once or twice each year. You prescribe acyclovir.
What is the mechanism of action of acyclovir when treating this type of infection?
1

Acts on viral thymidine kinase
2

Inhibits viral DNA polymerase
3

Inhibits viral RNA polymerase
4

Enhances CD8+ T-cell function leading to immune destruction of the virally infected cells
5

Is a non-competitive inhibitor of viral DNA polymerase at the pyrophosphate binding site
Q/Q(M)-480139 Report a Problem

A patient presents with tingling and burning of their lateral upper lip. They report that blisters will form
which then crust and heal. She gets these episodes once or twice each year. You prescribe acyclovir.
What is the mechanism of action of acyclovir when treating this type of infection?
2

Inhibits viral DNA polymerase
Acyclovir relies upon the fact that thymidine kinase is produced at a higher rate in herpes infected cells
than in noninfected cells. It is a guanosine analog that is preferentially phosphorylated by viral
thymidine kinase which then inhibits viral DNA polymerase, thus halting viral DNA synthesis by chain
termination. Acyclovir does not inhibit viral RNA polymerase or boost immune destruction of infected
cells. It is not a non-competitive inhibitor of viral DNA polymerase at the pyrophosphate binding site
this mechanism of action is that of Foscarnet, also an antiviral active against HSV.
Q/Q(M)-480139 Report a Problem
Gray-green discoloration of the mid-portion of permanent teeth is a side effect of?
1

Doxycycline
2

Tetracycline
3

Minocycline
4

Fluoroquinolones
5

clindamycin
Q/Q(M)-474450 Report a Problem
Gray-green discoloration of the mid-portion of permanent teeth is a side effect of?
3

Minocycline
In contrast to tetracycline staining of the teeth, which occurs in childhood and produces a brown
discoloration along the gingival third, minocycline stains the permanent teeth in adults, with a gray-
green discoloration of the mid-portion of the tooth.
47

Q/Q(M)-474450 Report a Problem
Which one of the following sunscreens has an absorption spectrum primarily in the UVA range?
1

Padimate O
2

Octyl Salicylate
3

Parsol 1789 (butyl dibenzoylmethane)
4

PABA (para-aminobenzoic acid)
5

Cinnamates
Q/Q(M)-477366 Report a Problem
Which one of the following sunscreens has an absorption spectrum primarily in the UVA range?
3

Parsol 1789 (butyl dibenzoylmethane)
Avobenzone (Parsol 1789) is primarily a UVA blocker. Photostability of avobenzone may be a problem
if it is combined with octyl methoxycinnamate. Salicylates, PABA, Padimate O, and cinnamates are
primarily UVB blockers.
Q/Q(M)-477366 Report a Problem
Tazarotene is what category for safety in pregnancy?
1

Category A
2

Category B
3

Category C
4

Category D
5

Category X
Q/Q(M)-474482 Report a Problem
Tazarotene is what category for safety in pregnancy?
5

Category X
Category X drugs include: acitretin, etretinate, estrogens, finasteride, 5-fluorouracil, flutamide,
isotretinoin, methotrexate, stanozolol, thalidomide, and tazarotene.
Q/Q(M)-474482 Report a Problem
Hemorrhagic cystitis is a risk of what chemotherapy?
1

Chlorambucil
2

Cyclophosphamide
3

Vinca alkaloids
4

Hydroxyurea
5

Azaithioprine
Q/Q(M)-474481 Report a Problem
Hemorrhagic cystitis is a risk of what chemotherapy?
2

Cyclophosphamide
Bladder toxicity is due to the acrolein metabolite of cyclophosphamide. Hemorrhagic cystitis is
associated with the increased risk of transitional cell carcinoma of the bladder. Mesna has been used to
reduce the toxic effect. The risk of cystitis is avoided by adequate fluid intake, frequent voiding, and
careful screening for hematuria.
48

Q/Q(M)-474481 Report a Problem
For which of the following medications is abdominal cramping and watery diarrhea a very common side
effect that may limit treatment?
1

Colchicine
2

Gold
3

Potassium iodide
4

Thalidomide
5

Chlorambucil
Q/Q(M)-475922 Report a Problem

For which of the following medications is abdominal cramping and watery diarrhea a very common side
effect that may limit treatment?
1

Colchicine
Colchicine is an alkaloid with antimitotic activity that is used in dermatology for its effects on
neutrophils. The most common side effect from colchicine use is gastrointestinal distress with
abdominal cramping and watery diarrhea.
Q/Q(M)-475922 Report a Problem
What is the recommended period for contraception after cessation of acitretin therapy in the United
States?
1

1 month
2

3 months
3

1 year
4

2 years
5

3 years
Q/Q(M)-474120 Report a Problem


What is the recommended period for contraception after cessation of acitretin therapy in the United
States?
5

3 years
Etretinate has a prolonged half-life of 80-160 days. The levels may persist up to 3 years in the body.
Acitretin can be converted to etretinate in the presence of ethanol.
Q/Q(M)-474120 Report a Problem

A 56 year-old man presents with blue-gray discoloration on his face, ears, and dorsal hands. What is the
most likely offending agent?
1

Minocycline
2

Amiodarone
3

Chloroquine
4

Quinacrine
49

5

Clofazimine
Q/Q(M)-477726 Report a Problem
A 56 year-old man presents with blue-gray discoloration on his face, ears, and dorsal hands. What is the
most likely offending agent?
2

Amiodarone
The patient presents with blue-gray discoloration in sun-exposed areas. The most likely offending agent
is amiodarone. Blue-gray discoloration from minocycline usually occurs on legs. Chloroquine usually
causes blue-gray discoloration in the sclerae, teeth, buccal mucosa, nail beds, and pretibial areas.
Quinacrine causes yellow discoloration of skin and conjunctiva. CLofazimine usualy causes a red-brown
discoloration.
Q/Q(M)-477726 Report a Problem
Which of the following medications is most likely to interefere with the efficacy of oral contraceptives?
1

Rifampin
2

Tetracycline
3

Doxycycline
4

Trimethoprim-sulfamethaxasole
5

Amoxicillin
Q/Q(M)-476723 Report a Problem


Which of the following medications is most likely to interefere with the efficacy of oral contraceptives?
1

Rifampin
Rifampin has been shown to decrease the efficacy of oral contraceptives. It is an inducer of cytochrome
p450 which increases the metabolism of hormones thereby decreasing the efficacy of oral
contraceptives. There is no clear decrease in oral contraceptive efficacy with concomitant use of
ampicillin, ciprofloxacin, clarithromycin, doxycyline, metronidzole, ofloxacin, or tetracycline.
Q/Q(M)-476723 Report a Problem

A patient taking daily prednisone is advised to switch to alternate day dosing to decrease the risk of:
1

Glaucoma
2

Aseptic bone necrosis
3

Cataracts
4

Adrenal crisis
5

Osteoporosis
Q/Q(M)-475860 Report a Problem

A patient taking daily prednisone is advised to switch to alternate day dosing to decrease the risk of:
4

Adrenal crisis
Long-term therapy with oral corticosteroids can result in numerous adverse effects, including elevated
risks of glaucoma, cataracts, hypertension, diabetes, osteoporosis, adrenal axis suppression, and aseptic
bone necrosis. Alternate-day dosing or oral corticosteroids lowers the rate of adrenal axis suppression. It
is hypothesized that during the off day, cell mediated immunity, white blood cells subset levels, and
50

potassium excretion are normalized while the anti-inflammatory benefits of the drug persist. Alternate-
day corticosteroid therapy should be employed once adequate disease control has been attained with
daily dosing. Of note, the risk of cataracts, osteoporosis, and other adverse effects of long-term
corticosteroid use are not minimized with alternate-day dosing.
Q/Q(M)-475860 Report a Problem
Which of the following is an adverse effect of oral retinoid therapy?
1

Mania
2

Fibroydysplasia ossicficans progressiva
3

Vascular calcification
4

Hypertension
5

Hypertrichosis
Q/Q(M)-478616 Report a Problem
Which of the following is an adverse effect of oral retinoid therapy?
2

Fibroydysplasia ossicficans progressiva
Adverse effects of oral retinoid therapy: Cutaneous: Xerosis, palmoplantar digital desquamation,
retinoid dermatitis, photosensitivity, pyogenic granulomas, stickiness sensation, Staph aureus infxns
Hair: Teolgen effluvian, abnormal hair texture, dryness Nails: Fragility with nail softening, paronychia,
onycholysis Occular: Dry eyes w/visual blurriness, Blepharoconjunctivitis, Photophobia Oral: Cheilitis,
dry mouth, sore mouth and tongue Nasal: Nasal mucosal dryness, decreased mucous secretion, epistaxis
Musculoskeletal: Arthralgias, myalgias, fatigue and muscle weakness, tendinitis Neurological:
Headache, mild depression Gatrointestinal: N/V/D, adb pain
Q/Q(M)-478616 Report a Problem
Which of the following retinoid medications is a second-generation synthetic retinoid?
1

Acitretin
2

Isotretinoin
3

Tretinoin
4

Adapalene
5

Bexarotene
Q/Q(M)-482754 Report a Problem

Which of the following retinoid medications is a second-generation synthetic retinoid?
1

Acitretin
First generation synthetic retinoids include tretinoin and isotretinoin. Second generation synthetic
retinoids are etretinate and acitretin. Tazarotene, adapalene, and bexarotene are third generation.
Q/Q(M)-482754 Report a Problem
Which of the following has been associated with a lichenoid drug eruption?
1

Sulfasalazine
2

Hydrochlorothiazide
3

Acetaminophen
4

Erythromycin
5

Nicotinamide
Q/Q(M)-474329 Report a Problem
51

Which of the following has been associated with a lichenoid drug eruption?
2

Hydrochlorothiazide
Lichen-planus-like (lichenoid) drug eruptions have been reported with: antimalarials, ?-blockers,
captopril, gold, penicillamine, HCTZ, NSAIDs. Lichenoid drug reactions are often photodistributed.
Q/Q(M)-474329 Report a Problem

Thryoid function tests should be checked before and during therapy with which of the following
medications?
1

Colchicine
2

Gold
3

Potassium iodide
4

Thalidomide
5

Azathioprine
Q/Q(M)-475919 Report a Problem
Thryoid function tests should be checked before and during therapy with which of the following
medications?
3

Potassium iodide
The Wolff-Chaifkoff effect, which is the inhibition of thyroid hormone synthesis from excess iodides
which block organic iodides from binding in the thyroid, can be observed in patients on potassium
iodide therapy. In patients with normal thyroid function, autoregulatory mechanisms allow for
appropriate escape from this effect. In patients with impaired autoregulatory mechanisms, the Wolff-
Chaikoff effect can lead to hypothyroidism.
Q/Q(M)-475919 Report a Problem
One of the main concerns for prescribing azathioprine to a gout patient with a normal level of thiopurine
methyltransferase on chronic allopurinol is:
1

Azathioprine does not work
2

Allopurinol does not work
3

Patient may develp photosensitivity
4

Patient may become pancytopenic
5

No concerns
Q/Q(M)-477423 Report a Problem
One of the main concerns for prescribing azathioprine to a gout patient with a normal level of thiopurine
methyltransferase on chronic allopurinol is:
4

Patient may become pancytopenic
Allopurinol inhibits xanthine oxidase, an enzyme that catabolizes azathioprine. In patients concurrently
taking bothe medications, the allopurinol shunts more 6-MP from the xanthine oxidase catabolic
pathway to the hypoxanthine-guanine phosphoribosyltransferase(HGPRT)anabolic pathway, creating an
excess of purine analogs. This in turn may lead to to excessive immunosuppression and risk of
pancytopenia.
Q/Q(M)-477423 Report a Problem
The form of erythromycin most likely to cause jaundice is:
1

Ethylsuccinate
52

2

Estolate
3

Stearate
4

Gluceptate
5

Lactobionate
Q/Q(M)-478604 Report a Problem
The form of erythromycin most likely to cause jaundice is:
2

Estolate
The form of erythromycin that most likely causes jaundice is estolate.
Q/Q(M)-478604 Report a Problem
Bone marrow suppression can occur more often in individuals taking azathioprine with genetically low
allele activity of what enzyme?
1

Inosine monophosphate dehydrogenase
2

Thiopurine methyltransferase
3

Glucose-6-phosphatase
4

Dihydrofolate reductase
5

Epoxide hydroxylase
Q/Q(M)-474467 Report a Problem
Bone marrow suppression can occur more often in individuals taking azathioprine with genetically low
allele activity of what enzyme?
2

Thiopurine methyltransferase
Thiopurine methyltransferase is one of three enzymes that metabolize azathioprine along with xanthine
oxidase and HGPRT. Decreased activity of TPMT leads to heightened metabolism by HGPRT into the
active 6-thioguanine. This may lead to increased toxicity including bone marrow suppression.
Q/Q(M)-474467 Report a Problem
Which of the following best describes the contents of a cream?
1

A semisolid emulsion of oil in water
2

A semisolid transparent emulsion
3

An aqueous or alcohol-based substance that may contain a salt in solution
4

A semisolid grease/oil, sometimes also containing powder, but little or now water
5

An ointment with a high proportion of powder
Q/Q(M)-482382 Report a Problem
Which of the following best describes the contents of a cream?
1

A semisolid emulsion of oil in water
A cream is a semisolid emulsion of oil in water that contains a preservative to prevent overgrowth of
microorganisms. A gel is a semisolid transparent, nongreasy emulsion. A lotion is a liquid vehicle that is
aqueous or alcohol-based that may contain a salt in solution. An ointment is a semisolid grease/oil,
sometimes also containing powder, but little or now water; usually there is no preservative needed. A
paste is an ointment with a high proportion of powder that gives it a stiff consistency.
Q/Q(M)-482382 Report a Problem
The combination of doxorubicin and which medications has been reported to cause sticky skin?
1

Vancomycin
53

2

Amphotercin
3

Ketoconazole
4

G-CSF
5

Cisplatin
Q/Q(M)-476715 Report a Problem
The combination of doxorubicin and which medications has been reported to cause sticky skin?
3

Ketoconazole
Polsen et. al. reported a 29% incidence of sticky skin in patients treated with high dose ketoconazole and
doxorubicin for prostate cancer. Other medications reported to cause this include etretinate and tretinoin.
Polsen JA, Cohen PR, Sella: Acquired cutaneous adherence in patients with androgen-independent
prostate cancer receiving ketoconazole and doxorubicin: medication-induced sticky skin. J Am Acad
Dermatol: 32 (4):571-5 1995
Q/Q(M)-476715 Report a Problem
A 21-year old male presents with a sudden onset of fever, rash, facial edema, leukocytosis and hepatitis
four weeks after starting phenytoin for seizures. This patient most likely had which risk factor for
developing this skin condition:
1

Slow acetylator
2

Deficiency of epoxide hydroxylase
3

Renal failure
4

Recent vaccination
5

Thyroid disease
Q/Q(M)-482273 Report a Problem

A 21-year old male presents with a sudden onset of fever, rash, facial edema, leukocytosis and hepatitis
four weeks after starting phenytoin for seizures. This patient most likely had which risk factor for
developing this skin condition:
2

Deficiency of epoxide hydroxylase
This patient has anticonvulsant hypersensitivity syndrome most likely resulting from a deficiency of
epoxide hydroxylase. Slow acetylators are also predisposed to this condition after being given
sulfonamides. Renal failure is a risk factor with allopurinol administration. Recent vaccination and
thyroid disease are not known risk factors.
Q/Q(M)-482273 Report a Problem
What antifungal is known to cause gynecomastia and impotence?
1

Griseofulvin
2

Itraconazole
3

Terbinafine
4

Ketoconazole
5

Fluconazole
Q/Q(M)-474453 Report a Problem
What antifungal is known to cause gynecomastia and impotence?
4

Ketoconazole
Ketoconazole is known to cause gynecomastia and impotence, by interfering with androgen and
glucocorticoid synthesis.
54

Q/Q(M)-474453 Report a Problem
Which one of the following agents when used prophylactically has demonstrated efficacy in reducing
skin cancer in organ transplant patients?
1

Tacrolimus
2

Acitretin
3

Imiquimod
4

Interleukin-2
5

Cyclosporine
Q/Q(M)-477198 Report a Problem
Which one of the following agents when used prophylactically has demonstrated efficacy in reducing
skin cancer in organ transplant patients?
2

Acitretin
Systemic retinoids like acitretin have been used in the chemoprophylaxis of skin cancers. Several studies
have suggested a beneficial effect of retinoids with lowered incidences of skin cancer. However,
following cessation of the retinoids, the incidence normalizes.
Q/Q(M)-477198 Report a Problem

Which of the following drugs has been known to cause penile erosions?
1

Gancyclovir
2

Cidofovir
3

Foscarnet
4

Penciclovir
5

Abacavir
Q/Q(M)-475200 Report a Problem
Which of the following drugs has been known to cause penile erosions?
3

Foscarnet
Foscarnet has been reported to cause penile erosions.
Q/Q(M)-475200 Report a Problem
The agent of choice used to acutely lower methemoglobin levels in patients taking dapsone is:
1

Cimetidine
2

Oral methylene blue
3

Homocysteine
4

Vitamin E
5

Glucose-6-phosphatase
Q/Q(M)-474466 Report a Problem

The agent of choice used to acutely lower methemoglobin levels in patients taking dapsone is:
2

Oral methylene blue
Cimetidine and vitamin E have both been known to provide prophylaxis against methemoglobin
formation. G6PD-deficient individuals are at greater risk of hematologic toxicity from dapsone.
55

Q/Q(M)-474466 Report a Problem

A patient with AIDS is given combination anti-retroviral therapy. Which of the following antiviral
medications used for HIV is associated with lipodystrophy with abnormal fatty deposits?
1

Indinavir
2

Abacavir
3

Didanosine
4

Zidovudine
5

Foscarnet
Q/Q(M)-482745 Report a Problem

A patient with AIDS is given combination anti-retroviral therapy. Which of the following antiviral
medications used for HIV is associated with lipodystrophy with abnormal fatty deposits?
1

Indinavir
Indinavir is a protease inhibitor used for the treatment of HIV. The protease enzyme is responible for the
final assembly of new viral proteins. The protease inhibitors, particularly indinavir, have been associated
with lipodystrophy, which manifests as abnormal fatty deposits known as the \"buffalo hump\" and
\"protease pouch.\"
Q/Q(M)-482745 Report a Problem

Which antifungal medication has a black box warning for CHF?
3

Itraconazole
Itraconazole should be avoided in heart patients due to its black box warning against CHF. Other
reported adverse effects from itraconazole include photosensitivity and COPD.
Q/Q(M)-482458 Report a Problem

An 87 year old female with chronic lymphocytic leukemia develops disseminated varicella-zoster
infection. She is hospitalized for treatment. Rapid intravenous infusion of acyclovir has been associated
with what complication?
1

Reversible obstructive nephropathy
2

Disseminated intravascular coagulation
3

Thrombocytopenia
4

Pulmonary fibrosis
5

Serum sickness
Q/Q(M)-482743 Report a Problem

An 87 year old female with chronic lymphocytic leukemia develops disseminated varicella-zoster
infection. She is hospitalized for treatment. Rapid intravenous infusion of acyclovir has been associated
with what complication?
1

Reversible obstructive nephropathy
Acyclovir is a guanosine analog. It is preferentially phosphorylated by viral thymidine kinase. It inhibits
viral DNA polymerase, halting viral DNA synthesis by chain termination. Rapid intravenous infusion of
acyclovir has been associated with a reverisble obstructive nephropathy.
56

Q/Q(M)-482743 Report a Problem

The antihistamine with strong H1 and H2 blockade is:
1

Chlorpheniramine
2

Cetirizine
3

Cimetidine
4

Cyproheptadine
5

Doxepin
Q/Q(M)-474479 Report a Problem

The antihistamine with strong H1 and H2 blockade is:
5

Doxepin
Doxepin, a tricyclic antidepressant, has H1 and H2 antihistamine activity.
Q/Q(M)-474479 Report a Problem

Methemoglobinemia is an adverse reaction to treatment with which agent?
1

Dapsone
2

Methotrexate
3

Plaquenil
4

Azathioprine
5

Cyclophosphamide
Q/Q(M)-475871 Report a Problem
Methemoglobinemia is an adverse reaction to treatment with which agent?
1

Dapsone
Dapsone is a lipid-soluble sulfone drug that is used widely in dermatology in a variety of conditions
including dermatitis herpetiformis, leprosy, and neutrophilic dermatoses. Adverse effects from dapsone
are both pharmacologic and idiosyncratic and include hemolytic anemia, methemoglobinemia,
agranulocytosis, hypersensitivity syndrome and neuropathy. Of these, the first two are pharmacologic
and anticipated, to some degree, in most patients treated with dapsone. However, the magnitude of
toxicity varies greatly among individuals on the drug. Methemoglobinemia is the formation of
methemoglobin in the blood, which has a decreased oxygen-carrying capacity compared with
hemoglobin and can result in cyanosis. The reaction is related to the N-hydroxy metabolites of dapsone,
which are potent oxidants. G6PD-deficient individuals are more susceptible to oxidative stresses,
including those from dapsone metabolites, and a baseline G6PD level is recommended prior to initiation
of dapsone therapy. In the event of emergent methemoglobinemia, oral methylene blue (100-300
mg/day) can be used to decrease methemoglobin levels. However, if the patient is G6PD deficient, this
strategy is ineffective.
Q/Q(M)-475871 Report a Problem
Clinical evidence of hypothyroidism can be induced by which drug?
1

Bexarotene
2

Gold
57

3

Griseofulvin
4

Acitretin
5

Isotretinoin
Q/Q(M)-474128 Report a Problem

Clinical evidence of hypothyroidism can be induced by which drug?
1

Bexarotene
Bexarotene can cause a central hypothyroidism with low TSH and T4.
Q/Q(M)-474128 Report a Problem

A 59 year-old woman is diagnosed with acute myelogenous leukemia. Induction chemotherapy is
initiated. A few days later the patient develops tender erythematous plaques on her face. What is the
most likely culprit drug?
1

Methotrexate
2

Cytarabine
3

5-fluorouracil
4

Cyclophosphamide
5

Hydroxyurea
Q/Q(M)-477675 Report a Problem

A 59 year-old woman is diagnosed with acute myelogenous leukemia. Induction chemotherapy is
initiated. A few days later the patient develops tender erythematous plaques on her face. What is the
most likely culprit drug?
2

Cytarabine
The most likely diagnosis is neutrophilic eccrine hidradenititis. Cytarabine is the usual offending agent.
Q/Q(M)-477675 Report a Problem

What is the most common cause of nonpigmented fixed drug eruption?
1

Naproxen
2

Tetracycline
3

Sulfonamides
4

Phenolphthalein
5

Pseudoephedrine
Q/Q(M)-477446 Report a Problem

What is the most common cause of nonpigmented fixed drug eruption?
5

Pseudoephedrine
Pigmented incontinence is usually prominent in a fixed drug eruption; yet occasionally, fixed drug
reactions do not result in long-lasting hyperpigmentation. The so-called nonpigmented fixed drug
eruption is distictive. Pseudoephedrine hydrochloride is by far the most common cause.
Q/Q(M)-477446 Report a Problem

58

Which of the following statements is not true regarding the tetracycline antiobiotics?
1

Ingestion of zinc salts may impair absorption of tetracycline
2

Tetracycline is the most common cause of fixed drug eruption
3

Tetracycline is more phototoxic than demeclocycline
4

These antibiotics are effective against Mycoplasma infections
5

Tetracyclines are contraindicated in children less than 9 years of age
Q/Q(M)-475194 Report a Problem

Which of the following statements is not true regarding the tetracycline antiobiotics?
3

Tetracycline is more phototoxic than demeclocycline
Demeclocycline and doxycycline are the most phototoxic of all the tetracyclines.
Q/Q(M)-475194 Report a Problem

Which cutaneous side effect is a common complication of nitrogen mustard therapy?
1

Bullous drug eruption
2

Telangiectasia
3

Hyperpigmentation
4

Allergic contact dermatitis
5

Fixed drug eruption
Q/Q(M)-477136 Report a Problem
Which cutaneous side effect is a common complication of nitrogen mustard therapy?
4

Allergic contact dermatitis
Topical nitrogen mustard, or mechlorethamine, is an antineoplastic agent which works via alkylation
thereby inhibiting DNA synthesis. Allergic contact dermatitis occurs in two-thirds of patients who are
treated with topical nitrogen mustard in aqueous solution, but occurs in less than 5% of patients treated
with the ointment based preparation.
Q/Q(M)-477136 Report a Problem

Which of the following side effects has not been reported in association with intravenous immune
globulin?
1

Headache
2

Flushing
3

Hypotension
4

Stevens-Johnson syndrome
5

Anaphylaxis
Q/Q(M)-477186 Report a Problem
Which of the following side effects has not been reported in association with intravenous immune
globulin?
4

Stevens-Johnson syndrome
IVIG is used to treat several diseases including graft versus host disease, connective tissue disease, and
59

autoimmune bullous dermatoses. Adverse effects include infusion reactions (headache, flushing, chills,
myalgia, wheezing, achycardia, lower back pain, nausea, or hypotension). Anaphylaxis occurs rarely.
Disseminated intravascular coagulation, transient neutropenia, and aseptic meningitis syndrome has
been reported. Cutaneous adverse effects include eczematous eruptions and alopecia.
Q/Q(M)-477186 Report a Problem

A 44 year old female with HIV/AIDS develops grouped painful vesicles in a T4 dermatomal
distribution. Valacyclovir is prescribed. What severe reaction has been associated with the use of
valacyclovir in AIDS patients?
1

Thrombotic thrombocytopenia purpura/hemolytic uremic syndrome
2

Stevens-Johnson syndrome
3

Acute renal failure
4

Jarisch-Herxheimer reaction
5

Disseminated intravascular coagulation
Q/Q(M)-482744 Report a Problem
A 44 year old female with HIV/AIDS develops grouped painful vesicles in a T4 dermatomal
distribution. Valacyclovir is prescribed. What severe reaction has been associated with the use of
valacyclovir in AIDS patients?
1

Thrombotic thrombocytopenia purpura/hemolytic uremic syndrome
Valacyclovir is a prodrug of acyclovir, and has enhanced bioavailability and converts rapidly and
completely to acyclovir. Severe and even fatal cases of thrombotic thrombocytopenia purpura/hemolytic
uremic syndrome have been reported in AIDS and transplant patients taking high doses of valacyclovir.
Q/Q(M)-482744 Report a Problem
After exposure to a tick, a patient develops a rickettsial infection, requiring a tetracycline-class
antibiotic. A review of systems reveals that he has a history of chronic renal insufficiency. Which is the
only tetracycline that can be used in patients with renal failure?
1

Doxycycline
2

Minocycline
3

Tetracycline
4

Demeclocycline
5

Azithromycin
Q/Q(M)-482740 Report a Problem
After exposure to a tick, a patient develops a rickettsial infection, requiring a tetracycline-class
antibiotic. A review of systems reveals that he has a history of chronic renal insufficiency. Which is the
only tetracycline that can be used in patients with renal failure?
1

Doxycycline
Tetracycline antibiotics inhibit protein synthesis by binding to the 30S ribosomal subunit. They are
effective against gram positive and negative organisms, Mycoplasma, Chlamydia, Rickettsia, and others.
Doxycycline, which is excreted by the Gi tract, is the only tetracycline for use in patients with renal
failure.
Q/Q(M)-482740 Report a Problem
Which of the following is not associated with calcipotriene?
60

1

Inactivation by basic pH
2

Increase in interleukin-10
3

Decrease in interleukin-2
4

Potential risk for photosensitivity
5

Inhibition of NF-kB
Q/Q(M)-482269 Report a Problem
Which of the following is not associated with calcipotriene?
1

Inactivation by basic pH
Calcipotriene is relatively unstable and inactivated by acidic pH, ie with ammonium lactate and salicylic
acid. The other statements are true. Psoriasis is classically a TH1 disease and this medicine upregulates
TH2 cytokines it is commonly used to treat this condition. It has mild anti-inflammatory properties via
its inhibition of NF-kB. It also inhibits cell proliferation and induces cell differentiation.
Q/Q(M)-482269 Report a Problem
Which of the following medications is associated with painful symmetrical erythema and paresthesias of
the palms and soles?
1

sorafenib
2

cetuximab
3

bortezomib
4

erlotinib
5

gefitinib
Q/Q(M)-482519 Report a Problem
Which of the following medications is associated with painful symmetrical erythema and paresthesias of
the palms and soles?
1

sorafenib
Sorafenib is a small-molecule multi-kinase inhibitor used in the treatment of renal cell cancer and
hepatocellular cancer. Notably it inhibits Raf kinase, which functions in the Ras signaling pathway, in
addition to blocking VEGFR-2, VEGFR-3, FLT3, and PDGFRb signaling. Reports of hand and foot
syndrome have been noted, characterized by acral erythema and paresthesias. This side effect seems to
resolve rapidly after discontinuation of treatment.
Q/Q(M)-482519 Report a Problem
Alternate-day administration of oral steroids can reduce all of the following side effects except?
1

Growth impairment
2

HPA axis suppression
3

Cataracts
4

Peptic ulcer disease
5

Opportunisitic infection
Q/Q(M)-474455 Report a Problem
Alternate-day administration of oral steroids can reduce all of the following side effects except?
61

3

Cataracts
Alternate-day corticosteroid dosing regimens does not decrease the risks of posterior subcapsular
cataracts, osteoporosis, and possibly osteonecrosis.
Q/Q(M)-474455 Report a Problem
All of the following statements are true regarding cyclosporin A EXCEPT:
1

Adverse effects include hypertrichosis and gingival hyperplasia
2

The most common eletrolyte abnormalities are hypokalemia and hypermagnesemia
3

Forms a complex with cyclophilin, blocking its ability to activate calcineurin, thus preventing
calcineurin from de-phosphorylation NFAT-1
4

Metabolized by the hepatic cytochrome P-450 3A4 enzyme system
5

NSAIDs can potentiate renal toxicity when combined with cyclosporine
Q/Q(M)-475611 Report a Problem

All of the following statements are true regarding cyclosporin A EXCEPT:
2

The most common eletrolyte abnormalities are hypokalemia and hypermagnesemia
The most common electrolyte abnormalities associated with cyclosporin A are hyperkalemia,
hyperuricemia, and hypomagnesemia.
Q/Q(M)-475611 Report a Problem



All of the following retinoids are excreted in the urine EXCEPT:
1

Bexarotene
2

Etretinate
3

Isotretinoin
4

Tretinoin
5

Acitretin
Q/Q(M)-475618 Report a Problem
All of the following retinoids are excreted in the urine EXCEPT:
1

Bexarotene
Bexarotene is excreted via hepatobiliary excretion. The others are excreted in bile and urine.
Q/Q(M)-475618 Report a Problem
At standard dosages, which of the following is fungicidal?
1

Terbinafine
2

Fluconazole
3

Ketoconazole
4

Itraconazole
5

Griseofulvin
Q/Q(M)-476687 Report a Problem
At standard dosages, which of the following is fungicidal?
62

1

Terbinafine
Terbinafine block ergosterol synthesis early in the synthetic pathway by inhibitins squalene epoxidase.
Squalene then accumulates within fungal cells and discupts cell membranes. At standard dosaging, it is
believed to be fungicidal. The other choices are fungistatic.
Q/Q(M)-476687 Report a Problem
The most specific marker of drug-induced lupus is:
1

Anti-histone Ab
2

ANA
3

Anti-Ro Ab
4

Anti-La Ab
5

Anti-ds DNA Ab
Q/Q(M)-474310 Report a Problem
The most specific marker of drug-induced lupus is:
1

Anti-histone Ab
Anti-histone Ab is most specific for drug-induced lupus. In addition to minocycline, hydralazine,
procainamide, isonaizid (INH), penicillamine and anti-convulsants have been associated with drug-
induced lupus-like syndrome.
Q/Q(M)-474310 Report a Problem
Which antiparasitic agent is highly flammable?
1

Lindane
2

Permethrin
3

Precipitated sulfur
4

Thiabendazole
5

Malathion
Q/Q(M)-475910 Report a Problem
Which antiparasitic agent is highly flammable?
5

Malathion
Malathion, an organophosphate cholinesterase inhibitor used to treat scabies and head lice, is flammable.
Q/Q(M)-475910 Report a Problem
Side effects from this antihistamine include gynecomastia, impotence, and loss of libido:
1

Doxepin
2

Cyproheptadine
3

Promethazine
4

Fexofenadine
5

Cimetidine
Q/Q(M)-475913 Report a Problem
Side effects from this antihistamine include gynecomastia, impotence, and loss of libido:
5

Cimetidine
Cimetidine, an H2 antihistamine, also competitively inhibits dihydrotestosterone at the androgen
63

receptor site, with resultant antiandrogen side effects including gynecomastia, impotence, and loss of
libido.
Q/Q(M)-475913 Report a Problem
The risk of pseudotumor cerebri in patients taking isotretinoin is increased by:
1

Dehydration
2

Concomitant use of tetracycline
3

Concomitant use of TMP-SMX
4

Doses higher than 1.0 mg/kg/day
5

Comorbid affective disorder
Q/Q(M)-474324 Report a Problem
The risk of pseudotumor cerebri in patients taking isotretinoin is increased by:
2

Concomitant use of tetracycline
The risk of pseudotumor cerebri is increased in patients on isotretinoin and a tetracycline.
Q/Q(M)-474324 Report a Problem
Which cell type is increased by glucocorticoids?
1

Neutrophils
2

Monocytes
3

T-cells
4

B-cells
5

Eosinophils
Q/Q(M)-474454 Report a Problem
Which cell type is increased by glucocorticoids?
1

Neutrophils
Glucocorticoids alter the balance of circulating leukocytes, causing an increase in the number of
polymorphonuclear leukocytes and diminishing the numbers of lymphocytes, eosinophils, and
monocytes.
Q/Q(M)-474454 Report a Problem

Which of the following vehicles is correctly defined
1

Ointment-water in oil emulsion
2

Gel-oil in water emulsion
3

Cream-Semisolid emulsion in alcohol base
4

Lotion/Solution-Cream in water
5

Foam - powder in cream
Q/Q(M)-478612 Report a Problem


Which of the following vehicles is correctly defined
1

Ointment-water in oil emulsion
Various vehicles are used for different clinical situations. Ointments are water in oil emulsions. Creams
64

are oil in water emulsion. Gels are semisolid emulsion in alcohol base . Lotions/Solutions are powder in
water (some oil). Foams are liquid and/or solid materials in a gaseous medium. e. All are correct (false)
Q/Q(M)-478612 Report a Problem
What family of medications is associated with xerosis?
1

Sulfonylureas
2

Beta blockers
3

Calcium channel blockers
4

Loop diuretics
5

Cholesterol lowering agents
Q/Q(M)-476690 Report a Problem


What family of medications is associated with xerosis?
5

Cholesterol lowering agents
Medications that alter the lipid composition of the epidermis and stratum corneum may impair the
normal barrier function of the skin. Cholesterol lowering medications like HMG-CoA reductase
inhibitors and niacin may cause xerosis through this mechanism.
Q/Q(M)-476690 Report a Problem
The steroid with the least minerocorticoid activity is:
1

Hydrocortisone
2

Cortisone
3

Prednisone
4

Methylprednisolone
5

Prednisolone
Q/Q(M)-474122 Report a Problem
The steroid with the least minerocorticoid activity is:
4

Methylprednisolone
Of the corticosteroids listed, the steroid with the lowest mineralcorticoid activity is methylprednisolone.
Minerocorticoids act on the kidney to decrease the rate of sodium excretion (with accompanying
retention of water). Triamcinolone, dexamethasone, and betamethasone also have low mineralcorticoid
activity.
Q/Q(M)-474122 Report a Problem
Cyclosporin A should not be consumed with grapefruit juice due to:
1

Induction of CYP2D6 by grapefruit juice
2

Inhibition of CYP2D6 by grapefruit juice
3

Induction of CYP3A4 by grapefruit juice
4

Inhibition of CYP3A4 by grapefruit juice
5

Grapefruit juice binds cyclosporine, inactivating it in the GI tract.
Q/Q(M)-480137 Report a Problem
Cyclosporin A should not be consumed with grapefruit juice due to:
65

4

Inhibition of CYP3A4 by grapefruit juice
Grapefruit juice is an inhibitor of CYP3A4 at the intestinal mucosal membrane. The intestinal CYP3A4
is involved in "first pass" metabolism - thus with inhibition of this enzyme in the gut, less cyclosporine
is metabolized, allowing for greater absorption of cyclosporine. Saquinivir also can be affected by this
"first pass" inhibition by grapefruit juice.
Q/Q(M)-480137 Report a Problem
The mechanism action of this cytotoxic agent is via inhibition of IMP dehydrogenase.
1

Azathioprine
2

Methotrexate
3

Hydroxyurea
4

5-fluorouracil
5

Mycophenolate mofetil
Q/Q(M)-477676 Report a Problem
The mechanism action of this cytotoxic agent is via inhibition of IMP dehydrogenase.
5

Mycophenolate mofetil
Mycophenolate mofetil (cellcept), a purine analog, blocks de novo purine synthesis by inhibiting the
enzyme inosine monophosphate dehydrogenase.
Q/Q(M)-477676 Report a Problem
Over use of of which medication may lead to this clinical image?
1

Topical steroid
2

Topical antibiotic
3

Calcipotriene
4

Imiquimod
5

Hydroquinone
Q/Q(M)-476871 Report a Problem


Over use of of which medication may lead to this clinical image?
5

Hydroquinone
Exogenous ochronosis has been reported with prolonged use of high concentration hydroquinone. On
pathology, a characteristic ochre colored deposit is noted between the collagen bundles.
Q/Q(M)-476871 Report a Problem
What feature best distinguishes lichen planus from lichenoid drug eruption?
1

segmental hypergranulosis
66

2

parakeratosis
3

eosinophils
4

spongiosis
5

saw tooth pattern
Q/Q(M)-482857 Report a Problem
What feature best distinguishes lichen planus from lichenoid drug eruption?
2

parakeratosis
Lichenoid drug reaction and lichen planus are best distinguished clinically. However, there are clues to
lichenoid drug, including parakeratosis and eosinophils. The presence of parakeratosis has been shown
to be more sensitive than eosinophils in the diagnosis of lichenoid drug reaction.
Q/Q(M)-482857 Report a Problem
Which of the following is a potentially irreversible ocular side effects of antimalarial agents?
1

Corneal deposition - causing halos, blurred vision, photophobia
2

Neuromuscular eye toxicity
3

Retinopathy
4

Cataracts
5

Pterygium
Q/Q(M)-477440 Report a Problem

Which of the following is a potentially irreversible ocular side effects of antimalarial agents?
3

Retinopathy
Three types of ocular adverse effects may develop from antimalarials: corneal deposits, neuromuscular
eye toxicity and retinopathy. Only retinopathy is potentially irreversible. It is recommended that a
patient be evaluated for retinopathy at baseline, then every 6 months by an ophthamologist. Testing
visual acuity, visual fields and performing a funduscopic examination are considered acceptable for
screening purposes.
Q/Q(M)-477440 Report a Problem
Which of the following statements regarding the ocular toxicities of the antimalarial drugs is NOT true?
1

Premaculopathy associated with changes in visual fields without visual loss is reversible if the
antimalarial is discontinued
2

True retinopathy is associated with "bull's eye" pigment deposition, central scotoma, and
diminished visual acuity
3

Risk of retinopathy is greatest for quinacrine, followed by chloroquine
4

Chloroquine and hydroxychloroquine should not be given together because of an additive effect
on retinotoxicity
5

The 4-aminoquinolones may have significant associated ocular toxicity
Q/Q(M)-475613 Report a Problem
Which of the following statements regarding the ocular toxicities of the antimalarial drugs is NOT true?
3

Risk of retinopathy is greatest for quinacrine, followed by chloroquine
Risk of retinopathy is greatest with chloroquine and does not exist for quinacrine.
Q/Q(M)-475613 Report a Problem
67

A 15 year old boy presents with a 4 month history of pigmented bands on several fingernails and
toenails. The most like etiology is:
1

Peutz-Jeghers syndrome
2

Chloroquine therapy
3

Minocycline therapy
4

Nevomelanocytic nevi
5

Acral lentiginous melanoma
Q/Q(M)-477321 Report a Problem
A 15 year old boy presents with a 4 month history of pigmented bands on several fingernails and
toenails. The most like etiology is:
3

Minocycline therapy
Melanonychia occurring simultaneously on several nails is most likely to be due to minocycline therapy.
Blue-black pigmentation may be present in nails, skin, scars and sclerae.
Q/Q(M)-477321 Report a Problem

Which drug has mucocutaneous side effects which can include stomatitis, chelitis, lichen planus-like
eruptions, and pityriasis rosea-like eruptions?
1

Colchicine
2

Gold
3

Potassium iodide
4

Thalidomide
5

None of the above
Q/Q(M)-475918 Report a Problem
Which drug has mucocutaneous side effects which can include stomatitis, chelitis, lichen planus-like
eruptions, and pityriasis rosea-like eruptions?
2

Gold
Mucocutaneous side effects, which are more common with injectable gold, include stomatitis, cheilitis,
lichen planus-like eruptions, and pityriasis rosea-like eruptions.
Q/Q(M)-475918 Report a Problem

Which antifungal is fungicidal?
1

Itraconazole
2

Terbinafine
3

Fluconazole
4

Clotrimazole
5

Griseofulvin
Q/Q(M)-482332 Report a Problem
Which antifungal is fungicidal?
2

Terbinafine
Terbinafine is an allylamine antifungal that is fungicidal. It works via noncompetitive inhibition of
squalene epoxydation. Amphotericin is also fungicidal. The azoles are fungistatic and inhibit 14-alpha-
68

demthylase to prevent ergosterol synthesis. Griseofulvin disrupts microtubule formation.
Q/Q(M)-482332 Report a Problem
Which of the following oral agents has been effective in the treatment of Norwegian scabies?
1

Thiabendazole
2

Ivermectin
3

Mebendazole
4

Griseofulvin
5

Metroniddazole
Q/Q(M)-475856 Report a Problem
Which of the following oral agents has been effective in the treatment of Norwegian scabies?
2

Ivermectin
Ivermectin (Stromectol) is an anti-helminthic agent currently FDA-approved for the treatment of
strongyloides and onchocerciasis. Several publications have reported efficacy of this agent in the
treatment of scabies and head lice. Due to its low rate of adverse effects, its high rate of effectiveness,
and its ease of administration, some authors consider this agent to be the treatment of choice for scabies
and head lice. The mechanism of action of ivermectin is blockade of glutamate-gated, chloride ion
channels, with adverse effects on nerve and muscle resulting in paralysis and death of the helminth or
mite. The drug has a very low affinity for mammalian chloride channels resulting in its relatively low
toxicity. Adverse effects are rare and have been associated with accidental intoxication. It should be
avoided when there is compromise of the blood-brain-barrier. Ivermectin is pregnancy category C.
There are virtually no associated drug interactions with oral ivermectin therapy.
Q/Q(M)-475856 Report a Problem

Dihydroxyacetone is found in which of the following products?
1

Rubber accelerators
2

Shampoos
3

Artificial nails
4

Hair dyes
5

Sunless tanning preparations
Q/Q(M)-482295 Report a Problem
Dihydroxyacetone is found in which of the following products?
5

Sunless tanning preparations
Dihydroxyacetone is the active ingredient in sunless tanning preparations. Upon oxidation it turns
orange-brown and binds to the stratum corneum. Rubber accelerators contain mercaptobenthothiazole,
carba mix, thiuram mix, or mercapto mix. Shampoos, especially "tear-free" ones, may contain
cocamidopropyl betaine. Artificial nails may contain methyl methacrylate or ethyl acrylate. Hair dyes
often have paraphenylenediamine.
Q/Q(M)-482295 Report a Problem
What testing can be performed to asses for methotrexate induced liver fibrosis, in lieu of liver biopsy?
1

AST/ALT ratio
2

Alkaline phosphatase level
3

procollagen 3 testing
69

4

procollagen 1 testing
5

procollagen 4 testing
Q/Q(M)-482847 Report a Problem
What testing can be performed to asses for methotrexate induced liver fibrosis, in lieu of liver biopsy?
3

procollagen 3 testing
procollagen 3 testing. Procollagen 3 has been shown to be a sensitive marker for methotrexate induced
liver fibrosis. Procollagen 1 is a marker for AFX.
Q/Q(M)-482847 Report a Problem
Neutrophilic eccrine hidradenitis is a side effect of which therapeutic agent?
1

Interferon-alpha
2

Cytarabine
3

Interferon type I
4

Intravenous immune globulin
5

Granulocyte colony stimulating factor
Q/Q(M)-477318 Report a Problem
Neutrophilic eccrine hidradenitis is a side effect of which therapeutic agent?
2

Cytarabine
Neutrophilic eccrine hidradenitis most commonly occurs in the setting of a patient with acute
myelogenous leukemia being treated with cytarabine. Clinical manifestations include tender,
erythematous macules, papuls and plaques on the trunk, neck and extremities which resolve within a few
days. Histologically, this drug eruption is defined by the presence of dense neutrophilic infiltrate within
and around eccrine glands, with necrosis of eccrine epithelial cells.
Q/Q(M)-477318 Report a Problem

What is the half-life of isotretinoin?
1

100 days
2

100 hours
3

50 hours
4

20 hours
5

10 hours
Q/Q(M)-475614 Report a Problem
What is the half-life of isotretinoin?
4

20 hours
The half-life of isotretinoin is 20 hours.
Q/Q(M)-475614 Report a Problem
Which of the following medicines reduces the formation of methemoglobin?
1

Dapsone
2

Cimetidine
3

Azathioprine
70

4

Cyclosporine
5

Rifampin
Q/Q(M)-482268 Report a Problem
Which of the following medicines reduces the formation of methemoglobin?
2

Cimetidine
Cimetidine and vitamin E can reduces the formation of methemoglobin, thereby increasing tolerance to
dapsone which can lead to methemoglobinemia. Azathioprine can cause hepatitis, lymphoproliferative
malignancy, infections, and hypersensitivity syndrome. Cyclosporine can lead to renal dysfunction,
hypertension, and ginvigal hyperplasia. Rifampin can cause decreased effectiveness of oral
contraceptives.
Q/Q(M)-482268 Report a Problem
Which of the following medications can lead to hematologic toxicity when combined with
methotrexate?
1

Trimethoprim
2

Sulfonamides
3

Dapsone
4

All of these answers are correct
5

None of these answers are correct
Q/Q(M)-475208 Report a Problem
Which of the following medications can lead to hematologic toxicity when combined with
methotrexate?
4

All of these answers are correct
All of the above inhibit the folic acid metabolic pathway, and can lead to hematologic toxicity when
combined with methotrexate.
Q/Q(M)-475208 Report a Problem
Which of the following cytotoxic agents has been associated with poikiloderma of the dorsal hands with
a band-like distribution of the fingers and toes?
1

Intralesional bleomycin
2

Doxorubicin
3

Flurouracil
4

Azathioprine
5

Hydroxyurea
Q/Q(M)-475207 Report a Problem
Which of the following cytotoxic agents has been associated with poikiloderma of the dorsal hands with
a band-like distribution of the fingers and toes?
5

Hydroxyurea
Hydroxyurea has been associated with poikiloderma of the dorsal hands with a band-like distribution of
the fingers and toes. It has also been associated with diffuse hyperpigmentation, and with leg ulcers
upon withdrawal.
Q/Q(M)-475207 Report a Problem
Imiquimod has proven beneficial in the treatment of all of the following lesions except:
71

1

Erythroplasia of Queyrat
2

Superficial basal cell carcinoma
3

Actinic keratoses
4

Squamous cell carcinoma
5

Extramammary Pagets disease
Q/Q(M)-477226 Report a Problem
Imiquimod has proven beneficial in the treatment of all of the following lesions except:
4

Squamous cell carcinoma
Imiquimod is an immune response modifier that acts via the toll like receptor 7. It induces cytokines
including interferon alpha, interleukins 1, 5, 6, 8, 10, and tumor necrosis factor. It has been used to treat
Erythroplasia of Queyrat, superficial basal cell carcinomas, actinic keratoses, extramammary Pagets
disease, and warts.
Q/Q(M)-477226 Report a Problem
All of the following are true regarding water-soluble retinoids EXCEPT:
1

They are undetectable in the serum after 1 month of stopping therapy
2

They include isotretinoin
3

They include etretinate
4

They include bexarotene
5

They have very little lipid deposition
Q/Q(M)-475617 Report a Problem
All of the following are true regarding water-soluble retinoids EXCEPT:
3

They include etretinate
Isotretinoin, acitretin, and bexarotene are water-soluble, with very little lipid deposition. Etretinate is 50
times more lipophilic than acitretin, with increased storage in adipose tissue.
Q/Q(M)-475617 Report a Problem
Which of the following antiparasitic agents is an organophosphate cholinesterase inhibitor?
1

Precipitated sulfur
2

Thiabendazole
3

Ivermectin
4

Lindane
5

Malathion
Q/Q(M)-475908 Report a Problem
Which of the following antiparasitic agents is an organophosphate cholinesterase inhibitor?
5

Malathion
Malathion is an organophosphate cholinesterase inhibitor.
Q/Q(M)-475908 Report a Problem
The Wolff-Chaikoff effect is associated with what medication?
1

Bexarotene
2

Zidovudine
72

3

Potassium iodide
4

Hydroxychloroquine
5

Thalidomide
Q/Q(M)-474480 Report a Problem
The Wolff-Chaikoff effect is associated with what medication?
3

Potassium iodide
The Wolff-Chaikoff effect is the inhibition of thyroid hormone synthesis from excess iodides which
block organic iodides from binding in the thyroid. In patients with normal thyroid function,
autoregulatory mechanisms allow for escape from this effect. In patients with impaired autoregulatory
mechanisms, the Wolff-Chaikoff effect can lead to hypothyroidism. Thyroid function should be
evaluated and monitored with patients started on potassium iodide.
Q/Q(M)-474480 Report a Problem
In regards to liposuction, what is the maximum amount of tumescent anesthesia (lidocaine)
recommended for an adult?
1

15 mg/kg
2

25 mg/kg
3

35 mg/kg
4

45 mg/kg
5

55 mg/kg
Q/Q(M)-482346 Report a Problem
In regards to liposuction, what is the maximum amount of tumescent anesthesia (lidocaine)
recommended for an adult?
5

55 mg/kg
Technically, 35 mg/kg is considered the safe and average amount recommended. However, maximum
amount of lidocaine has been reported to be 55 mg/kg. Tumescent anesthesia is a method where dilute
lidocaine is injected into subcutaneous fat for the liposuction procedure.
Q/Q(M)-482346 Report a Problem

The SPF of a sunscreen is based on applying the sunscreen at what concentration?
1

1 mg/cm2
2

2 mg/cm2
3

3 mg/cm2
4

4 mg/cm2
5

5 mg/cm2
Q/Q(M)-477493 Report a Problem
The SPF of a sunscreen is based on applying the sunscreen at what concentration?
2

2 mg/cm2
A sunscreen SPF is based on using it at a concentration of 2 mg/cm2 which is about 1 ounce or 30 grams
for the entire average sized body. It also is about 3-5 grams for the head and neck.
Q/Q(M)-477493 Report a Problem
Which of the following chemotherapeutic agents is not cell cycle specific?
1

Methotrexate
73

2

5-fluorouracil
3

Cyclophosphamide
4

Azathioprine
5

Hydroxyurea
Q/Q(M)-482195 Report a Problem
Which of the following chemotherapeutic agents is not cell cycle specific?
3

Cyclophosphamide
Cyclophosphamide is an alkylating agent that cross links DNA at any point during the cell cycle.
Methotrexate, 5-fluorouracil, azathioprine, and hydroxyurea are S-phase specific cytotoxic agents.
Methotrexate is a folic acid analog that blocks dihydrofolate reductase. 5-FU is a pyrimidine analog that
prevents the conversion of deoxyuridine monophosphate to deoxythymidine monophosphate in DNA
synthesis. Azathioprine is a purine analog that is converted into the active metabolite 6-thioguanine bit
hypoxanthine guanine phosphoribosyltransferase. Hydroxyurea inhibits ribonucleotide reductase which
normally converts ribonucleotides to deoxyribonucleotides in DNA synthesis.
Q/Q(M)-482195 Report a Problem
Which of the following statements regarding sunscreens is true?
1

Physical blockers absorb ultraviolet light and convert it to lower energy wavelengths
2

Methyl anthranilate is a UVB absorber
3

Padimate O is a UVB absorber
4

Photoallergy has not been reported to benzophenones
5

PABA and its derivates do not cross react with sulfonamides
Q/Q(M)-475899 Report a Problem
Which of the following statements regarding sunscreens is true?
3

Padimate O is a UVB absorber
Physical blockers reflect and scatter UV rays, whereas chemical sunscreens absorb UV light and convert
the absorbed energy into longer lower energy wavelengths. Methyl anthranilate is a UVA blocker.
Padimate O, a PABA derivative, is a UVB blocker. Photoallergy has been reported with increasing
frequency to benzophenones. Allergic contact allergy can occur with PABA and its derivatives, which
can cross react with azodyes, aniline, procaine, benzocaine, paraphenylenediamine, and sulfonamides.
Q/Q(M)-475899 Report a Problem
Which of the following medications is associated with anti-myeloperoxidase antibodies?
1

Isoniazid
2

Minocycline
3

Penicillamine
4

TNF-alpha inhibitors
5

Griseofulvin
Q/Q(M)-482509 Report a Problem

Which of the following medications is associated with anti-myeloperoxidase antibodies?
2

Minocycline
Minocycline, which is widely used in the treatment of acne, often without adequate supervision, may
induce arthritis and livedo vasculitis associated with anti-MPO (pANCA). (O Elkayam. Ann Rheum Dis.
74

1996) Isoniazid most often causes drug-induced lupus associated with anti-hitone antibodies.
Penicillamine and TNF-inhibitors are associated with anti-dsDNA antibodies. Griseofulvin can
exacerbate lupus but is most often associated with drug-induced SCLE and anti-Ro antibodies.
Q/Q(M)-482509 Report a Problem

Which of the following is true regarding the use of cyclosporine?
1

Guidelines limit the continuous use of cyclosporine in the US to 5 years
2

Cyclosporine should not be used together with methotrexate
3

Cyclosporine is not an effective treatment for psoriatic arthritis
4

Cyclosporine inhibits the activation of antigen presenting cells
5

Cyclosporine has not shown efficacy in the treatment of chronic urticaria
Q/Q(M)-482511 Report a Problem
Which of the following is true regarding the use of cyclosporine?
4

Cyclosporine inhibits the activation of antigen presenting cells
Cyclosporine binds to cyclophilin, an intracellular immunophilin, and inhibits the activity of calcineurin
phosphatase, which is then unable to phosphorylate NFAT. Guidelines limit the use of cyclosporine in
the US to one year. Cyclosporine is an effective treatment for psoriatic arthritis, alone or in combination
with methotrexate. Cyclosporine inhibits the activation of T-cells, NK cells, and antigen presenting
cells. And cyclosporine has shown some efficacy in histamine resistant chronic urticaria.
Q/Q(M)-482511 Report a Problem
This patient also has a diagnosis of acne vulgaris (diagnosed 10 years ago). What is the most likely
explanation for these cutaneous changes?
1

Peripheral vascular disease
2

Diabetes mellitus
3

Drugs hypersensitivity reaction to penicillin
4

Minocycline-induced hyperpigmentation
5

Leukocytoclastic vasculitis
Q/Q(M)-480555 Report a Problem


This patient also has a diagnosis of acne vulgaris (diagnosed 10 years ago). What is the most likely
explanation for these cutaneous changes?
4

Minocycline-induced hyperpigmentation
This patient has minocyclin-induced hyperpigmentation. Hyperpigmentation is one of the most
75

commonly observed side effects of minocycline and occurs regardless of dosage or treatment duration
(although most often seen in patients after long-term treatment with the drug). Three distributions can be
seen: Type I (blue-black pigmentation in sites of inflammation/scars), Type II (blue-black pigmentation
on the anterior lower legs, and Type III (muddy brown diffuse pigmentation on normal, sun-exposed
skin). This patient has Type II pigmentation.
Q/Q(M)-480555 Report a Problem


Which member of the tetracycline family is most likely to have caused this photomediated reaction?
1

Demeclocycline
2

Doxycycline
3

Minocycline
4

Oxytetracycline
5

Tetracycline
Q/Q(M)-476862 Report a Problem


Which member of the tetracycline family is most likely to have caused this photomediated reaction?
1

Demeclocycline
The tetracycline family of antibiotics are bacteriostatic and act by inhibiting protein synthesis. Each
member of the family may cause photosensitivity, but demeclocycline is the most photosensitizing.
Q/Q(M)-476862 Report a Problem

Which of the following medications used in dermatology is known to increase the risk for bone
fractures?
1

Dapsone
2

Isotretinoin
3

Acitretin
4

Prednisone
5

Doxycycline
Q/Q(M)-482190 Report a Problem

Which of the following medications used in dermatology is known to increase the risk for bone
fractures?
4

Prednisone
76

Long term use of oral corticosteroids has been shown to decrease bone mineral density (BMD). This
decrease in BMD leads to an increased risk for fractures. It was originally thought that Vitamin A
analogues such as isotretinoin and acitretin may also increase the risk of fracture but in a large case-
controlled study published in the May 2010 Archives illustrates that risk of fracture is not associated
with Vitamin A analogues. Vitamin A analogues have been associated with hyperostosis.
Q/Q(M)-482190 Report a Problem


A child with systemic medium-vessel vasculitis is treated with a cytotoxic agent. While in the hospital,
he develops new-onset generalized tonic-clonic seizures. Which cytotoxic agent is associated with tonic-
clonic seizures?
1

Chlorambucil
2

Cyclophosphamide
3

Methotrexate
4

Doxorubicin
5

Hydroxyurea
Q/Q(M)-482748 Report a Problem
A child with systemic medium-vessel vasculitis is treated with a cytotoxic agent. While in the hospital,
he develops new-onset generalized tonic-clonic seizures. Which cytotoxic agent is associated with tonic-
clonic seizures?
1

Chlorambucil
Chlorambucil is a nitrogen mustard derivative that is used as a steroid sparing agent in vasculitis,
Behcet\'s disease, dermatomyositis, and sarcoidosis. In children with nephritic syndrome or adults with a
seizure history, it can cause generalized tonic-clonic seizures.
Q/Q(M)-482748 Report a Problem
Which drug is known to cause an SCLE-like eruption?
1

Captopril
2

Minocycline
3

Barbituates
4

Furosemide
5

Terbinafine
Q/Q(M)-474129 Report a Problem
Which drug is known to cause an SCLE-like eruption?
5

Terbinafine
SCLE-like reaction are known to occur most likely from glyburide, griseofulvin, hydrochlorothiazide,
penicillamine, piroxicam, and terbinafine.
Q/Q(M)-474129 Report a Problem
A 66 year old female presents with palpable purplish lesions and small necrotic ulcers of the extremities.
Additional workup reveals renal and hepatic involvement, and a diagnosis of Wegener\'s granulomatosis
is made. Which of the following cytotoxic agents is the treatment of choice for Wegener\'s
granulomatosis?
1

Cyclophosphamide
2

Chlorambucil
3

Methotrexate
77

4

Bleomycin
5

Mycophenolate mofetil
Q/Q(M)-482747 Report a Problem
A 66 year old female presents with palpable purplish lesions and small necrotic ulcers of the extremities.
Additional workup reveals renal and hepatic involvement, and a diagnosis of Wegener\'s granulomatosis
is made. Which of the following cytotoxic agents is the treatment of choice for Wegener\'s
granulomatosis?
1

Cyclophosphamide
Cyclophosphamide is a nitrogen mustard derivative. It is cell-cycle nonspecifc and works by producing
DNA cross-linkages at any point in the cell cycle. It is the treatment of choice for Wegener\'s
granulomatosis.
Q/Q(M)-482747 Report a Problem
A patient has recalcitrant Sweet\'s syndrome. Hydroxyurea is initiated. Hydroxyurea works by inhibiting
which enzyme?
1

Ribonucleotide reductase
2

DNA gyrase
3

Dihydrofolate reductase
4

Inosine monophosphate dehydrogenase
5

Thymidine kinase
Q/Q(M)-482751 Report a Problem

A patient has recalcitrant Sweet\'s syndrome. Hydroxyurea is initiated. Hydroxyurea works by inhibiting
which enzyme?
1

Ribonucleotide reductase
Hydroxyurea is a S-phase specific cytotoxic agent which inhibits ribonucleotide reductase, an enzyme
responsible for converting ribonucleotides to deoxyribonucleotides in DNA synthesis. Anemia, hepatitis,
and renal toxicity are associated adverse effects. Fluoroquinolones inhibit DNA gyrase. Methotrexate
inhibits dihydrofolate reductase. Mycophenolate mofetil inhibits inosine monophosphate dehydrogenase.
Acyclovir utitilizes thymidine kinase to interrupt viral replication.
Q/Q(M)-482751 Report a Problem
Weekly CD4 T-cell counts are recommended for psoriasis patients treated with which biologic agent?
1

Alefacept
2

Etanercept
3

Efalizumab
4

Infliximab
5

None of the above
Q/Q(M)-475914 Report a Problem
Weekly CD4 T-cell counts are recommended for psoriasis patients treated with which biologic agent?
1

Alefacept
Alefacept eliminates activated memory T-cells, so weekly CD4 T-cell counts are recommended.
Q/Q(M)-475914 Report a Problem
78

A 25 year old female patient has a recurrent pustule that occurs in the same place on the lower lip. The
drug that she can use is the drug that is the prodrug of acyclovir. It is used for the infection of HSV-1
and HSV-2 and can be used for recurrent infections:
1

Valacyclovir
2

Brivudin
3

Famciclovir
4

Griseofulvin
5

Dapsone
Q/Q(M)-482900 Report a Problem
A 25 year old female patient has a recurrent pustule that occurs in the same place on the lower lip. The
drug that she can use is the drug that is the prodrug of acyclovir. It is used for the infection of HSV-1
and HSV-2 and can be used for recurrent infections:
1

Valacyclovir
Valacyclovir is a prodrug of acyclovir and is used in acute HSV 1 and 2 infections. It is converted in
first pass through gastrointestinal tract and liver to acyclovir. It is probably better for zoster than
acyclovir. It is not approved in immunosuppressed patients.
Q/Q(M)-482900 Report a Problem
Which of the following inhibits bacterial cell wall synthesis by complexing with the carrier protein C55-
prenol pyrophosphatase.
1

Bacitracin
2

Polymyxin
3

Neomycin
4

Mupirocin
5

Silver sulfadiazine
Q/Q(M)-477441 Report a Problem
Which of the following inhibits bacterial cell wall synthesis by complexing with the carrier protein C55-
prenol pyrophosphatase.
1

Bacitracin
Bacitracin is a polypeptide antibiotic produced by the Tracey I stain of Bacillus subtilis. It inhibits
bacterial cell wall synthesis by complexing with the carrier protein C55-prenol pyrophosphatase, which
is involved in the transfer of polysaccharides, liposaccharides, and peptidoglycans to a growing cell
wall.
Q/Q(M)-477441 Report a Problem
What is a possible metabolic abnormality with long term systemic glucocorticosteroid therapy?
1

Hyperlipidemia
2

Hypoglycemia
3

Hyperkalemia
4

Hypercalcemia
5

Hypokalemic acidosis
Q/Q(M)-482831 Report a Problem
79

What is a possible metabolic abnormality with long term systemic glucocorticosteroid therapy?
1

Hyperlipidemia
Long term systemic steroid therapy can result in hyperlipidemia, especially Hypertriglyceridemia.
Hyperglycemia, not hypoglycemia may occur with steroid therapy. Hypokalemic alkalosis may result
from long term steroid use. Uncommonly, hypocalcemia may develop with resultant tetany.
Q/Q(M)-482831 Report a Problem
The treatment of choice for Wegner's granulomatosis is:
1

Chlorambucil
2

Systemic glucocorticosteroids
3

Cyclophosphamide
4

Methotrexate
5

Azaithioprine
Q/Q(M)-474468 Report a Problem

The treatment of choice for Wegner's granulomatosis is:
3

Cyclophosphamide
Wegener granulomatosis is an idiopathic disorder characterized by necrotizing granulomatous vasculitis
of the upper and lower respiratory tract, glomerulonephritis, and a variable degree of small-vessel
vasculitis. Testing for c-ANCA is 90% sensitive for the diagnosis when the presentation is classic.
Cyclophosphamide has been used very effectively and now is the usual drug of choice for induction of
remission.
Q/Q(M)-474468 Report a Problem
Treatment with isotretinoin has been shown to cause increased colonization of the skin with which of the
following organisms?
1

Streptococcus pyogenes
2

Pseudomonas aeruginosa
3

Staphylococcus aureus
4

Pityrosporum orbiculare
5

Demodex folliculorum
Q/Q(M)-477195 Report a Problem

Treatment with isotretinoin has been shown to cause increased colonization of the skin with which of the
following organisms?
3

Staphylococcus aureus
Staphylococcus aureus colonization tends to correlate with isotretinoin-induced reduction in sebum
production and may lead to infections. This complication may possibly be prevented with pulsed
intranasal mupirocen therapy. There has been a report of staphylococcus endocarditis in a patient with
underlying aortic insufficiency.
Q/Q(M)-477195 Report a Problem
Which of the following substances is not found in Castellani's paint?
1

Resorcinol
2

Acetone
3

Magenta
80

4

Phenol
5

Lactic acid
Q/Q(M)-482383 Report a Problem
Which of the following substances is not found in Castellani's paint?
5

Lactic acid
Castellani's paint is a fungicidal and bactericidal agent. It contains boric acid, resorcinol, acetone, water,
industrial methylated spirit, magenta, and phenol. Magenta may stain clothing and skin while phenol
may be toxic in children.
Q/Q(M)-482383 Report a Problem
Which drug has recently been shown to be of use in chemoprevention of squamous cell carcinoma in
recessive dystrophic epidermolysis bullosa:
1

Isotretinoin
2

Cyclosporine
3

Mycophenolate mofetil
4

Methotrexate
5

Topical tar
Q/Q(M)-478600 Report a Problem
Which drug has recently been shown to be of use in chemoprevention of squamous cell carcinoma in
recessive dystrophic epidermolysis bullosa:
1

Isotretinoin
Despite low therapeutic responses of advanced cancers to retinoids, these drugs appear to have a
promising role in chemoprevention. Patients with oral leukoplakia, actinic keratoses, arsenic keratoses,
and Bowen's disease can benefit from retinoid therapy.
Q/Q(M)-478600 Report a Problem

Which of the following antibiotics is the only drug that is bactericidal to Mycobacterium leprae?
1

Rifampin
2

Levofloxacin
3

Minocycline
4

Amikacin
5

Pentavalent antimony
Q/Q(M)-482742 Report a Problem

Which of the following antibiotics is the only drug that is bactericidal to Mycobacterium leprae?
1

Rifampin
Rifampin inhibits RNA synthesis by blocking DNA-dependent RNA polymerase. It is effective in
tuberculosis and atypical mycobacterial infections. It is the only drug that is bactericidal to
Mycobacterium leprae. Cutaneous leishmaniasis and rhinoscleroma also respond to rifampin.
Q/Q(M)-482742 Report a Problem
Methotrexate toxicity with myelosuppression is treated with what agent?
1

Folinic acid
81

2

Folic acid
3

Oral methylene blue
4

Cimetidine
5

Vitamin E
Q/Q(M)-474478 Report a Problem
Methotrexate toxicity with myelosuppression is treated with what agent?
1

Folinic acid
Leukovorin, or folinic acid, is able to bypass dihydrofolate reductase in the cell division pathway. It is
administered under conditions of methotrexate-induced myelosuppression. Leukovorin is also available
as an oral form, and may be administered as a continual part of methotrexate therapy, instead of folic
acid. It is now believed that neither folic acid nor folinic acid inhibit the efficacy of methotrexates
antipsoriatic actions.
Q/Q(M)-474478 Report a Problem
Which of the following statements regarding dapsone and sulfapyridine is true?
1

They exert their anti-inflammatory actions by stimulating the myeloperoxidase activity of
polymorphonuclear leukocytes
2

Dapsone hypersenstivity syndrome is characterized by lymphocytosis
3

Sulfapyridine has a similar but often more severe side effect profile than dapsone
4

Concomittant administration of cimetidine has been shown to increase the risk of
methemoglobinemia
5

None of these answers are correct (all are false)
Q/Q(M)-475612 Report a Problem
Which of the following statements regarding dapsone and sulfapyridine is true?
5

None of these answers are correct (all are false)
None of the above statements are true. Dapsone and sulfapyridine exert their anti-inflammatory actions
by inhibiting the myeloperoxidase activity and chemotactic abilities of polymorphonuclear leukocytes.
Dapsone hypersenstivity syndrome is characterized by eosinophilia, as well as a severe mononucleosis-
like reaction, including fever, erythroderma, hepatitis, and even death. Sulfapyridine has a similar but
often less severe side effect profile. Cimetidine has been shown to provide some protection against
methemoglobin formation.
Q/Q(M)-475612 Report a Problem
Which of the following statements regarding retinoids is true?
1

Tretinoin and isotretinoin are second-generation synthetic retinoids
2

Etretinate is a first-generation retinoid
3

Second-generation retinoids are polyaromatic retinoids
4

Tazarotene is a fourth-generation selective retinoid
5

Bexarotene is a third-generation retinoid
Q/Q(M)-475616 Report a Problem
Which of the following statements regarding retinoids is true?
5

Bexarotene is a third-generation retinoid
First-generation retinoids are tretinoin (all-trans RA) and isotretinoin (13-cis RA). Second-generation
synthetic retinoids are etretinate, which was replaced by its metabolite acitretin. Third-generation
82

(polyaromatic retinoids) include the arotinoids, tazarotene, adapalene, and bexarotene.
Q/Q(M)-475616 Report a Problem
Ketoconazole and itraconazole are potent inhibitors of which one of the following cytochrome p450
isozymes?
1

2C9
2

2D6
3

3A4
4

1A2
5

2C18
Q/Q(M)-478602 Report a Problem
Ketoconazole and itraconazole are potent inhibitors of which one of the following cytochrome p450
isozymes?
3

3A4
Ketoconazole has been shown to be the strongest inhibitor of cytochrome p450 (CYP) 3A4. Itraconazole
is an inhibitor of CYP 3A4, whereas fluconazole inhibites CYP 2C9 significantly more than minimal
inhibitory role of CYP 3A4. Terbinafine inhibits CYP 2D6.
Q/Q(M)-478602 Report a Problem
The treatment of choice for erythema nodosum leprosum (ENL) is:
1

Thalidomide
2

Clofazamine
3

Rifampin
4

Rifampin and clofazamine
5

Isoniazid, rifampin and clofazamine
Q/Q(M)-474464 Report a Problem
The treatment of choice for erythema nodosum leprosum (ENL) is:
1

Thalidomide
Erythema Nodosum Leprosum is a type 2 reaction of leprosy. It most commmonly ocurrs in lepromatous
leprosy. Clinically, it presents as painful nodules and is associated with fever, malaise, anorexia, and
arthralgias. The treatment of choice is thalidomide.
Q/Q(M)-474464 Report a Problem
Of the medications listed below, the safest to use during pregnancy is:
1

Penicillin
2

Erythromycin estolate
3

Doxycycline
4

Estrogens
5

NSAIDS
Q/Q(M)-474117 Report a Problem
Of the medications listed below, the safest to use during pregnancy is:
1

Penicillin
Erythromycin estolate is contraindicated in pregnancy because of the risk of cholestatic hepatitis. Other
83

forms of erythromycin are safer for use in pregnancy. Tetracyclines are category D and estrogens are
category X. NSAIDS may promote persistent fetal circulation or oligohydramnios.
Q/Q(M)-474117 Report a Problem
Which antihistamine has suppressor T-cell inhibitory activity?
1

Doxepin
2

Promethazine
3

Fexofenadine
4

Cimetidine
5

Cromolyn sodium
Q/Q(M)-475912 Report a Problem
Which antihistamine has suppressor T-cell inhibitory activity?
4

Cimetidine
Cimetidine is an H2 antihistamine that has suppressor T-cell inhibitory activity, by competitively
blocking their H2 receptors. Immunomodulatory effects are useful for treating mucocutaneous
candidiasis, verruca vulgaris, and condyloma acuminata.
Q/Q(M)-475912 Report a Problem
Which of the following bones will develop the most dramatic changes in density following oral
glucocorticoids?
1

Femur of 72 year old female
2

Femur of 72 year old male
3

Vertebrae of 22 year old male
4

Vertebrae of 22 year old female
5

Vertebrae of 72 year old female
Q/Q(M)-482224 Report a Problem
Which of the following bones will develop the most dramatic changes in density following oral
glucocorticoids?
3

Vertebrae of 22 year old male
Although post menopausal women are at greatest risk for fractures from steroid therapy (they have the
weakest bones to begin with), it is young men who have the most dramatic decrease in bone density.
Young men have the highest bone density on average and therefore will show the greatest change with
steroid therapy. Trabecular bone (ribs, vertebrae) are more affected than cortical bones (long bones) due
to higher metabolic rate of the former. Also, most demineralization occurs during the first 6-12 months
of therapy. This highlights the fact that even young men who are not traditionaly at risk for fractures,
need prophylaxis for osteroperosis while on systemic steroids.
Q/Q(M)-482224 Report a Problem
Which medication should be avoided in pregnant women with acne?
1

Isotretinoin
2

Azelaic acid
3

Clindamycin lotion
4

Erythromycin gel
5

Azithromycin
Q/Q(M)-482812 Report a Problem
84

Which medication should be avoided in pregnant women with acne?
1

Isotretinoin
Isotretinoin is absolutely contraindicated in pregnancy and is category X. All other alternatives are safe
in pregnancy.
Q/Q(M)-482812 Report a Problem
Which of the following locations has the highest percutaneous absorption of topical corticosteroids?
1

Scrotum
2

Axilla
3

Back
4

Arm
5

Palm
Q/Q(M)-482832 Report a Problem
Which of the following locations has the highest percutaneous absorption of topical corticosteroids?
1

Scrotum
Absorption through the scrotum is about 42 times greater than the ventral forearm. Although less, other
occluded sites such as the axilla have increased percutaneous absorption. The palms and soles have the
least due to the thick stratum corneum.
Q/Q(M)-482832 Report a Problem
Which one of the following sites has the highest percutaneous absorption of topical drugs?
1

Scrotum
2

Eyelid
3

Chest
4

Acral area
5

Mucous membrane
Q/Q(M)-482293 Report a Problem


Which one of the following sites has the highest percutaneous absorption of topical drugs?
5

Mucous membrane
Mucous membranes have the highest absorption of topical medicines. Percutaneous absorption, from
highest to lowest, is as follows: mucous membrane > scrotum > eyelids > face > chest and back > upper
extremity > lower extremity > acral.
Q/Q(M)-482293 Report a Problem
Which of the following antifungal agents is contraindicated in patients with a history of porphyria?
1

Griseofulvin
2

Ketoconazole
3

Fluconazole
4

Itraconazole
5

Terfinafine
Q/Q(M)-475210 Report a Problem
85

Which of the following antifungal agents is contraindicated in patients with a history of porphyria?
1

Griseofulvin
Griseofulvin has been reported as a potential exacerbator of acute intermittent porphyria, and thus is
contraindicated in patients with a history of porphyria.
Q/Q(M)-475210 Report a Problem
Which of the following statements is true regarding the absorption of antibiotics?
1

Fluoroquinolone absorption is not altered by antacids
2

Tetracycline absorption is impaired by the ingestion of calcium products but not iron
3

Minocycline absorption is not significantly impaired by the ingestion of calcium products but it
should be taken on an empty stomach
4

Doxycycline absorption is impaired by the ingestion of dairy products and calcium
5

None of the above (all of the above statements are false)
Q/Q(M)-475199 Report a Problem
Which of the following statements is true regarding the absorption of antibiotics?
5

None of the above (all of the above statements are false)
Antacids decrease the absorption of fluoroquinolones and should be taken at least 2 hours after the drug.
Tetracycline absorption is impaired by the ingestion of dairy products, calcium, and iron or zinc salts.
Minocycline and doxycycline absorption is not impaired by the ingestion of those products; they may be
taken on an empty stomach or with food. Thus, all of the statements are false.
Q/Q(M)-475199 Report a Problem
Dapsone is used for a patient with dermatitis herpetiformis. Dapsone induces anti-inflammatory effects
by primarily inhibiting what type of cell?
1

Polymorphonuclear leukocyte
2

T cell lymphocyte
3

Macrophage
4

Mast cell
5

Langerhans cell
Q/Q(M)-482752 Report a Problem
Dapsone is used for a patient with dermatitis herpetiformis. Dapsone induces anti-inflammatory effects
by primarily inhibiting what type of cell?
1

Polymorphonuclear leukocyte
Dapsone is used to treat a wide variety of dermatoses, including dermatitis herpetiformis, bullous SLE,
erythema elevatum diunitum, Sweet\'s syndrome, and others. The anti-inflammtory effect of this
medication is most effective against neutrophils because of inhibition of myeloperoxidase activity and
chemotactic abilities.
Q/Q(M)-482752 Report a Problem
Side effects of PUVA include all of the following except:
1

Headache
2

Pruritus
3

Neutropenia
4

Insomnia
86

5

Nausea
Q/Q(M)-477337 Report a Problem
Side effects of PUVA include all of the following except:
3

Neutropenia
Side effects of psoralen with ultraviolet A light (PUVA) include side effects which are due to phototoxic
effects including pruritus, photoonycholysis, friction blisters, ankle edema and hypertrichosis. In
addition, there are adverse effects from methoxypsoralen including gastrointestinal and neurologic
effects, hepatotoxicity and exanthems. Neutropenia is not a described side effect of PUVA.
Q/Q(M)-477337 Report a Problem
Which of the following chemotherapeutic agents has been linked to acneiform eruptions?
1

Bleomycin
2

Cytarabine
3

Cisplatin
4

Doxorubicin
5

Cetuximab
Q/Q(M)-476711 Report a Problem
Which of the following chemotherapeutic agents has been linked to acneiform eruptions?
5

Cetuximab
Cetuximab is a chimeric anti-epidermal growth factor receptor antibody that is FDA approved to treat
advanced colorectal cancer. Acneiform eruptions have been reported to occur in up to 1/3 of patients.
Q/Q(M)-476711 Report a Problem
Which of the following antiviral agents is NOT phosphorylated by viral thymidine kinase?
1

Acyclovir
2

Famciclovir
3

Gancyclovir
4

Cidofovir
5

Valacyclovir
Q/Q(M)-475201 Report a Problem
Which of the following antiviral agents is NOT phosphorylated by viral thymidine kinase?
4

Cidofovir
Cidofovir is a nucleotide analogue antiviral agent. It does not require phosphorylation by virus, but is
converted by host cell kinases to a diphosphate.
Q/Q(M)-475201 Report a Problem
Which drug may increase levels of digoxin?
1

Amoxicillin
2

Ciprofloxacin
3

Erythromycin
4

Minocycline
5

Cephalexin
Q/Q(M)-475197 Report a Problem
Which drug may increase levels of digoxin?
87

3

Erythromycin
Erythromycin inhibits the cytochrome P-450 system, which may result in increased levels of digoxin,
among many other drugs.
Q/Q(M)-475197 Report a Problem
The treatment of choice for scabies in pregnant women is:
1

Malathion
2

1% permethrin
3

Thiabendazole
4

Precipitated sulfur 6%
5

Ivermectin
Q/Q(M)-474463 Report a Problem
The treatment of choice for scabies in pregnant women is:
4

Precipitated sulfur 6%
Many authors consider precipitated sulfur to be the drug of choice for the treament of scabies in
pregnant women, although there are no published studies for this indication (see Wolverton). Permethrin
of note is category B in pregnancy.
Q/Q(M)-474463 Report a Problem
Which of the following agents is NOT a UVA blocker?
1

Dioxybenzone
2

Amyl p-dimethylaminobenzoate
3

Dibenzoylmethane
4

Avobenzone
5

Red veterinary petrolatum
Q/Q(M)-475901 Report a Problem
Which of the following agents is NOT a UVA blocker?
2

Amyl p-dimethylaminobenzoate
Dioxybenzone is one of the benzophenones (as in oxybenzone and sulisobenzone), which are UVA
blockers. Amyl p-dimethylaminobenzoate is a UVB blocker. Dibenzoylmethane (avobenzone; Parsol
1789) is a UVA/UVB blocker. Red veterinary petrolatum is a UVA blocker.
Q/Q(M)-475901 Report a Problem
The laboratory abnormality most associated with cyclosporine is:
1

Hypermagnesemia
2

Hyponatremia
3

Hyperkalemia
4

Hypouricemia
5

Increased LDH
Q/Q(M)-474116 Report a Problem
The laboratory abnormality most associated with cyclosporine is:
3

Hyperkalemia
The laboratory abnormalities associated with cyclosporine are decreased magnesium, increased
potassium, and increased uric acid. Renal function and blood pressure must also be monitored closely in
88

patients using cyclosporine.
Q/Q(M)-474116 Report a Problem
A 40 year old man with acute promyelocytic leukemia develops scrotal ulcers 1 month after starting
therapy. All tests for STD causes of ulcers as well as CMV and EBV are negative. What is the most
likely cause?
1

All-trans retinoic acid
2

Daunorubicin
3

Methotrexate
4

6-mercaptopurine
5

Cytarabine
Q/Q(M)-482557 Report a Problem
A 40 year old man with acute promyelocytic leukemia develops scrotal ulcers 1 month after starting
therapy. All tests for STD causes of ulcers as well as CMV and EBV are negative. What is the most
likely cause?
1

All-trans retinoic acid
All-trans-retinoic acid (ATRA) has been shown to improve the outcome of patients with acute
promyelocytic leukemia (APL). However, various adverse effects of ATRA treatment have been noted,
such as scrotal and genital ulcers.
Q/Q(M)-482557 Report a Problem
All of the following agents exert their function in a cell-cycle specific manner except:
1

Cyclophosphamide
2

Methotrexate
3

Azathioprine
4

Hydroxyurea
5

5-fluorouracil
Q/Q(M)-474469 Report a Problem
All of the following agents exert their function in a cell-cycle specific manner except:
1

Cyclophosphamide
Cyclophosphamide is a cell-cycle nonspecific agent, which produces DNA cross-linkages at any point in
the cell cycle. Methotrexate, azaithioprine, and hydroxyurea are S-phase specific cytotoxic agents. 5-
fluorouracil is a cell-cycle specific pyrimidine antagonist.
Q/Q(M)-474469 Report a Problem

You might also like